HIMT Final

¡Supera tus tareas y exámenes ahora con Quizwiz!

In which registry would you expect to find an Injury Severity Score (ISS)? A. birth defects registry B. trauma registry C. transplant registry D. cancer registry

B. trauma registry **Trauma registries maintain databases on patients with severe traumatic injuries. The Abbreviated Injury Scale (AIS) reflects the nature of the injury and the severity (threat to life) by body system.The Injury Severity Score (ISS) is an overall severity measurement calculated from the AIS scores for the three most severe injuries of the patient. The AIS and ISS classify and describe the severity of injuries and can be used for reporting registry activity.

A physician instructed his outpatient coder to report multiple codes in order to try and increase reimbursement when a single combination code should normally be reported. What is this called? A. jamming B. unbundling C. overcoding D. upcoding

B. unbundling **Unbundling is reporting multiple codes to increase reimbursement when a single combination code should be reported. Upcoding is reporting codes that are not supported by documentation in the patient record for the purpose of increasing reimbursement. Overcoding is reporting codes for signs and symptoms in addition to the established diagnosis code. Jamming is routinely assigning an unspecified ICD-9-CM or ICD-10-CM disease code instead of reviewing the coding manual to select the appropriate code.

The decision was made to drop bills in three days not four. The change must be applied in the: A. request for proposal. B. report writer. C. system settings. D. tables.

C. system settings. **System settings control the behavior of an information system.

In which brain lobe is the processing of smell and hearing stimuli performed? A. parietal B. frontal C. temporal D. occipital

C. temporal **Each of the paired lobes of the brain lying beneath the temples, including areas concerned with the understanding of speech.

A basic concept of office layout and workflow is that the A. paper and employee move to a predetermined location. B. employee moves to the paper. C. office layout and workflow should be revised frequently. D. paper moves to the employee.

D. paper moves to the employee. **For the process to be efficient, the paper should move to the employee rather than employees having to chase the paper to complete their tasks.

HIPAA Privacy Rule provides patients with the ability to request an amendment to their health record, to access their health record and other A. addressable requirements. B. preemptions. C. required standards. D. patient rights.

D. patient rights. **The HIPAA Privacy Rule allows patients access to their own health record, the right to request an amendment and other rights

The data center containing the computer upon which the electronic health record is located was flooded. As a result, the EHR is inoperable. Identify the type of security that failed. A. transmission B. network C. administrative D. physical

D. physical **Physical security protects hardware from theft, natural disasters, loss, and more.

Which endocrine gland secretes melatonin, which controls the circadian rhythm of an individual? A. alpha cells of the pancreas B. adrenal medulla C. beta cells of the pancreas D. pineal

D. pineal **Melatonin is the hormone produced in the pineal gland in the center of the brain; it regulates the body's daily rhythms such as the sleep-wake cycle.

The Centers for Medicare and Medicaid Services (CMS) will make an adjustment to the MS-DRG payment for certain conditions that the patient was not admitted with, but were acquired during the hospital stay. Therefore, hospitals are required to report an indicator for each diagnosis. This indicator is referred to as A. a sentinel event. B. a hospital acquired condition. C. a payment status indicator. D. present on admission.

D. present on admission.

You have had a problem with duplicate medical record numbers in your facility's MPI. Now you are joining a health information exchange, so you will need to clean up your database and prevent the duplicates from happening again. If you want to find near matches as well as identical matches in your MPI, you should implement a _________ algorithm. A. deterministic B. fuzzy C. none of these answers apply D. probabilistic

D. probabilistic **Probabilistic algorithms look for transpositions, name changes, and other near matches.

A radiologist is asked to review a patient's CT scan that was taken at another facility. The modifier -26 attached to the code indicates that the physician is billing for what component of the procedure? A. confirmatory B. global C. technical D. professional

D. professional **Modifier -26 is a professional component and is reported when the provider either interprets test results or operates equipment for a procedure.

Your role requires you to determine who is a business associate so that you can get a business associate agreement signed. Identify who qualifies as a business associate. A. bulk food service provider B. security guards C. childbirth class instructor D. release of information company

D. release of information company **Business associates require protected health information in order to perform work for the covered entity.

The _______________ is a statement sent to the provider to explain payments made by third-party payers. A. acknowledgment notice B. attestation statement C. advance beneficiary notice D. remittance advice

D. remittance advice **Remittance advice (RA) documents go from the third-party payer to the Provider, while the explanation of benefits (EOB) goes from the third-party payer to the Beneficiary.

The director of a Health Information Department has asked the department supervisors to review and revise all job descriptions. The supervisor over coding and analysis has determined that the job description for the senior coders must be revised. The decision to revise the job description is probably due to a change in the A. number of coders. B. results of a recent performance appraisal conducted on one particular coder. C. competitor facility's job description for its coders. D. scope of coding responsibilities.

D. scope of coding responsibilities. **Only a change in the coder's responsibilities would constitute the rewriting of the job description.

To properly implement performance improvement (PI), organizations should ensure that all employees participate in an integrated, continuous PI program. This is known as A. quality management liaison group. B. organizational PI. C. shared vision. D. shared leadership.

D. shared leadership. **The team approach, or shared leadership, permits all members to take responsibility for the improvements that need to be made.

Sam Spade has been severely injured in an MVA because he was not wearing a seat belt. The organ in his body, situated at the upper left of his abdominal cavity, under the ribs, that is part of his lymphatic system has been ruptured, and he is bleeding internally. Sam needs a surgical procedure known as A. sigmoidoscopy. B. sequestrectomy. C. sialoadenectomy. D. splenectomy.

D. splenectomy. **The spleen (left of the upper part of stomach under ribs) plays multiple supporting roles in the body. It acts as a filter for blood as part of the immune system, old red blood cells are recycled, and platelets and white blood cells are stored there; it also helps fight certain kinds of bacteria that cause pneumonia and meningitis.

An effective information governance system should include all of the following principles except one: A. the principle of retention B. the principle of disposition C. the principle of availability D. the principle of interoperability

D. the principle of interoperability **The principles of retention, availability, and disposition are all important in supporting proper information governance across the organization. The principle of interoperability is more limited to the IT Department.

Which structure transports urine from the kidneys to the bladder? A. urethra B. vas deferens C. meatus D. ureter

D. ureter **The ureter is the duct that passes the urine from the kidney to the bladder; the urethra passes the urine from the bladder to the outside of the body.

Each patient is asked to provide his or her driver's license as part of the admission process. This is known as patient A. access control. B. authorization. C. authentication. D. verification.

D. verification. **Patient verification is ensuring that the patient is who they say they are.

A patient is admitted to the hospital for a coronary artery bypass surgery. Postoperatively, he develops a pulmonary embolism. The present on admission (POA) indicator is A. U = documentation is insufficient to determine if condition was present at the time of admission. B. Y = present at the time of inpatient admission. C. N = not present at the time of inpatient admission. D. W = provider is unable to clinically determine if condition was present at the time of admission.

N = not present at the time of inpatient admission. **The pulmonary embolism is an acute condition that was not present on admission because it developed after the patient was admitted and after the patient had surgery.

Patient Jamey Smith has been seen at Oceanside Hospital four times prior to this current encounter. Unfortunately, because of clerical errors, Jamey's information was entered into the MPI incorrectly on the four previous admissions and consequently has four different medical record numbers. The unit numbering system is used at Oceanside Hospital. In looking at the entries into the MPI, which medical record number should be used for all visits?Jamey's previous entries into the MPI are as follows: 09/03/15 Jamey SmithMR# 10361 03/10/16 Jamey Smith DoeMR# 33998 07/23/17 Jamie Smith DoeMR# 36723 11/15/18 Jamey S. DoeMR# 41369 A. 10361 B. 41369 C. 36723 D. 33998

A. 10361 **All medical records should be filed and merged under the initial medical record number that was #10361. The information should not be filed and merged under the subsequent medical record numbers.

A covered entity must act on a request for accounting disclosures no later than _____ days after its receipt and within ______ days of extension, as long as it notifies the individual in writing of the reasons for the delay and when the accounting will be made available. A. 60, 30 B. 30, 60 C. 45, 60 D. 45, 30

A. 60, 30 **The covered entity (CE) must act on the individual's request for an accounting of disclosures no later than 60 days after receipt of such request. If the CE is unable to provide the accounting of disclosures within the time required, the CE may extend the time by no more than 30 days to provide the request.

The nursing staff would most likely use which of the following to facilitate aggregation of data for comparison at local, regional, national, and international levels? A. ABC Codes B. SPECIALIST Lexicon C. READ Codes D. LOINC

A. ABC Codes **READ Codes merged to create the SNOMED Clinical Terminology system. SPECIALIST Lexicon is an English language lexicon containing many biomedical terms. LONIC is a database and universal standard for identifying medical laboratory observations. ABC codes consist of five-character, alphabetic strings that identify services, remedies, and/or supplies. Codes are followed by two-character code modifiers which identify the practitioner type who delivered the care. In healthcare, the combination of ABC codes with older medical code sets allows many essential business processes (such as forecasting, practice management, insurance billing, claims processing, and third-party reimbursement) to be fully automated. The codes also help digitize information to simplify data collection, analysis, and reporting. This improves the quality of data and accelerates the speed with which conclusions can be drawn from those data about what works in US healthcare and why

Based on CMS's DRG system, other systems have been developed for payment purposes. The one that classifies the non-Medicare population, such as HIV patients, neonates, and pediatric patients, is known as A. APR-DRGs. B. AP-DRGs. C. IR-DRGs. D. RDRGs.

A. APR-DRGs. **APR = all patient refined.

Select the circumstances that require patient authorization prior to disclosure. A. Bureau of Disability Determination B. public health activities C. health oversight activity D. workers' compensation

A. Bureau of Disability Determination **The Bureau of Disability Determination is the only option that is not allowed by law to be released without patient authorization.

The committee that is responsible for establishing the quality improvement priorities of the cancer program and for monitoring the effectiveness of quality improvement activities is the A. Cancer Committee. B. Medical Staff Committee. C. Quality Improvement Committee. D. Governing Board Committee.

A. Cancer Committee. **The Cancer Committee is the designated multidisciplinary body for the administrative oversight, development, and review of cancer care services at a facility. This committee communicates directly with the facility's medical board, and its activities and recommendations directly impact programs.

As the compliance officer for a large physician practice group, you are interested in researching the original requirements for meaningful use of certified EHRs for use in an upcoming presentation. You begin by googling A. HITECH Act. B. Health Care Quality Improvement Act. C. HIPAA. D. EMTALA.

A. HITECH Act. **The HITECH Act was signed into law in 2009 to promote the adoption and meaningful use of EHRs and health information technology.

Which data bank is a result of HIPAA legislation? A. Healthcare Integrity and Protection Data Bank B. Fraud and Abuse Data Bank C. Agency for Healthcare Research and Quality D. Privacy Information Breach Data Bank

A. Healthcare Integrity and Protection Data Bank **The Healthcare Integrity and Protection Data Bank (HIPDB) was established by the Health Insurance Portability and Accountability Act of 1996, Public Law 104-191. HIPDB's authorizing statute is also referred to as Section 1128E of the Social Security Act.

In 1987, OBRA helped shift the focus in long-term care to patient outcomes. As a result, core assessment data elements are collected on each SNF resident as defined in the A. MDS. B. Uniform Clinical Data Set. C. UHDDS. D. Uniform Ambulatory Core Data.

A. MDS. **OBRA mandates comprehensive functional assessments of long-term care residents using the Minimum Data Set (MDS) for Long-Term Care.

You need to analyze data on the types of care provided to Medicare patients in your geographic area by DRG. Which of the following would be most helpful? A. MEDPAR B. Vital Statistics C. RxNorm D. National Practitioner Data Bank

A. MEDPAR **The Medicare Provider Analysis and Review (MEDPAR) file is made up of acute-care hospital and skilled nursing facility (SNF) claims data for all Medicare claims. It consists of the following types of data: demographic data on the patient, data on the provider, information on Medicare coverage for the claim, total charges, charges broken down by specific types of services, ICD-10-CM diagnosis and procedure codes, and DRGs.The MEDPAR file is frequently used for research on topics such as charges for particular types of care and analysis by DRG. The limitation of the MEDPAR data for research purposes is that it only contains data about Medicare patients.

The Institute of Medicine (IOM) published report titled "To Err Is Human: Building a Safer Health System," heightened concern by the U.S. government and accrediting agencies. This led the Joint Commission to place emphasis on improving patient safety and sentinel event occurrences through its safety program, known as A. National Patient Safety Goals (NPSG). B. Health Care Quality Improvement Program (HCQIP). C. ORYX Initiative Set. D. Health Plan Employer Data & Information (HEDIS).

A. National Patient Safety Goals (NPSG). **The publication of "To Err Is Human: Building a Safer Health System" led the Joint Commission to place emphasis on improving patient safety and sentinel event occurrences through its safety program, known as National Patient Safety Goals (NPSG). Refer to the Joint Commission website: http://www.jointcommission.org/assets/1/6/2015_NPSG_HAP.pdf

An organization that reviews the facility's provision of health care services, respond to Medicare beneficiary complaints that have been filed, and take action by implementing quality of care improvements is known as a(n) ______________ A. Quality Improvement Organization (QIO) B. Total Quality Management Program C. Lean Management Model D. HVBP program

A. Quality Improvement Organization (QIO) **Quality Improvement Organizations review the facility's provision of health care services, respond to Medicare beneficiary complaints that have been filed, and take action by implementing quality of care improvements.

ICD-10-PCS procedure codes are used on which of the following forms to report services provided to a patient? A. UB-04 B. MDC 02 C. CMS-1491 D. CMS-1500

A. UB-04 **ICD-10-PCS codes report inpatient services and procedures. UB-04 are claim forms used by hospitals to request reimbursement for providing inpatient services and procedures.

In general, which of the following statements is correct? A. When federal and state laws conflict, valid federal laws supersede state laws. B. When federal and state laws conflict, valid state laws supersede federal laws. C. When federal and state laws conflict, valid local laws supersede federal and state laws. D. When federal and state laws conflict, valid corporate policies supersede federal and state laws.

A. When federal and state laws conflict, valid federal laws supersede state laws. **HIPAA grants individuals the right to access their protected health information (PHI). Even though the organization owns the record, the patient has a right to access their record with the exceptions noted in HIPAA. There are many states that grant individuals the right to access their own health information. However, if the state law does not provide individuals with the same degree of access that is afforded by HIPAA, the federal law will supersede the state law.

An elevated serum amylase would be characteristic of A. acute pancreatitis. B. postrenal failure. C. gallbladder disease. D. prerenal failure.

A. acute pancreatitis. **Blood amylase and lipase levels are most frequently drawn to diagnose pancreatitis; when the pancreas is inflamed, increased blood levels of amylase and lipase, the pancreatic enzymes, will result.

The minimum record retention period for patients who are minors is A. age of majority plus the statute of limitations. B. age of majority. C. 5 years past treatment. D. 2 years past treatment.

A. age of majority plus the statute of limitations. **The minimum record retention period for a minor is the age of majority plus the statute of limitations. A minor may file a lawsuit on his or her behalf upon reaching the age of majority. The statute of limitations does not begin to run until the minor reaches the age of majority.

A valid authorization for release of information contains A. all of these answers apply. B. the date and signature of the patient or the patient's authorized representative. C. the name, agency, or institution to which the information is to be provided. D. the name of the hospital or provider who is releasing the medical information.

A. all of these answers apply. **A valid authorization must contain at least the following elements: a description of the information to be used or disclosed that identifies specifics in a meaningful way; the specific identification of the person(s) authorized to make the request; specific identification to whom the covered entity may make the requested disclosure; a description of each purpose of the requested use or disclosure; an expiration date or expiration event; and the signature of the individual and date. It must also contain the individual's right to revoke the authorization in writing; the ability or inability for the authorization to place conditions on TPO, enrollment, or eligibility; and the potential for information disclosed pursuant the authorization to be subject to redisclosure.

The Cancer Committee at Wharton General Hospital wants to compare long-term survival rates for pancreatic cancer by evaluating medical versus surgical treatment of the cancer. The best source of these data is the A. cancer registry abstracts. B. disease index. C. master patient index. D. operation index.

A. cancer registry abstracts. **Abstracts organize, summarise, and categorize the crucial information in a patient's medical record. The cancer registry abstract contains information about each patient from the time of diagnosis and continuing throughout the patient's life.

The process of cancer development, from exposure through the cellular changes of hyperplasia to neoplasia, is termed A. carcinogenesis. B. pathogenesis. C. staging. D. metastasis.

A. carcinogenesis. ** Carcinogenesis is the process by which normal cells are transformed into cancer cells; it is a complex multistep process, which can be generally divided into three major stages: initiation, promotion, and progression.

When substituting a photocopy of the original record in response to legal process, which of the following can be helpful in convincing the court to accept the photocopy as a true and exact copy of the original? A. certificate of authentication B. consent from the patient C. consent from the hospital administrator D. correspondence from the attending physician

A. certificate of authentication **During a legal proceeding, parties must agree to allow a photocopy of the record to be introduced into evidence instead of the original. A written certification that the photocopy being provided is an exact copy is required.

In a hospital, a document that contains a computer-generated list of procedures, services, and supplies, along with their revenue codes and charges for each item, is known as a(n) A. chargemaster. B. superbill. C. revenue master. D. encounter form.

A. chargemaster. **Chargemaster is an organization's master list of all services that can be provided to patients.

Darling v. Charleston Community Memorial Hospital is considered one of the benchmark cases in health care because it was with this case that the doctrine of _______________ was eliminated for nonprofit hospitals. A. charitable immunity B. professional negligence C. corporate negligence D. contributory negligence

A. charitable immunity **Under the doctrine of charitable immunity, hospitals were traditionally protected from liability because of its classification as a charitable organization. This privilege was gradually eliminated through state legislation and case law. Darling v. Charleston Community Memorial Hospital was the landmark case responsible for significantly eliminating this protection. The courts have used the doctrine of corporate negligence and respondeat superior to establish liability.

Mary Smith, a 48-year-old patient, is receiving an IV mixture of four different medications to treat stage two invasive ductal breast carcinoma. Each of the medications acts upon a different aspect of the cancer cells. This mixture is typically termed a(n) A. cocktail. B. mashup. C. blend. D. amalgamation.

A. cocktail. **In order to kill several cancer cells as quickly as possible, several drugs are usually given by IV at one time. These mixtures of various liquid medications are typically referred to as cocktails.

What is the best tool for differentiating between common cause variation and special cause variation? A. control chart B. scatter diagram C. run chart D. Pareto chart

A. control chart **A control chart is used to study variations in performance over time, and its source.

Jane believes new equipment will make her staff more efficient. The board has asked for an analysis of the cost of the equipment to be compared with the benefit to the facility. This mathematical process is known as a ________ A. cost-benefit analysis B. force field analysis C. performance improvement plan D. Pareto

A. cost-benefit analysis **A cost-benefit analysis is a mathematical process by which to evaluate whether or not the purchase of something has a value equal to or greater than the amount paid.

The process of reviewing and validating qualifications, granting professional or medical staff membership, and awarding delineated privileges is called the A. credentialing. B. appointment. C. licensure. D. professional review.

A. credentialing. **It is the responsibility to confirm, from appropriate authorities, that the candidate has the proper credentials before granting privileges.

A retrospective review as part of quality improvement activities is conducted after the patient has been A. discharged. B. admitted. C. released from the surgical recovery room. D. cleared for surgery.

A. discharged. **Retrospective means past. Therefore, a retrospective review is conducted after the patient has been discharged.

Which of the following actions illustrates the use of employee empowerment by upper management? A. discussing suggested approaches of improving productivity and performance with employees B. providing supervision only when requested by employees C. basing decisions on information found in the manual of operations D. micromanaging all aspects and functions performed in the department

A. discussing suggested approaches of improving productivity and performance with employees **Empowered employees have the freedom to contribute ideas and perform their jobs in the best possible way.

Mandatory reporting requirements for vital statistics generally A. do not require authorization by the patient. B. do not apply to health care facilities. C. require authorization by the physician. D. require authorization by the payer.

A. do not require authorization by the patient. **Patients do not have the opportunity to agree or object to those disclosures that have a public interest and benefit. For example, reporting requirements as required by law, such as vital statistics, do not require a patient authorization.

A PEG procedure would most likely be done to facilitate A. eating. B. breathing. C. urination. D. none of these answers apply.

A. eating **Percutaneous endoscopic gastrostomy (PEG) is an endoscopic medical procedure in which a tube (PEG tube) is passed into a patient's stomach through the abdominal wall, most commonly to provide a means of feeding when oral intake is not adequate.

Health plans that use ____________ reimbursement methods issue lump-sum payments to providers to compensate them for all the health care services delivered to a patient over a specific period of time for a particular reason. A. episode of care (EOC) B. capitation C. bundled D. fee-for-service

A. episode of care (EOC) **Episode-of-care reimbursement is based on the standards of care based on the diagnosis.

In ICD-10-PCS, if a "resection of tonsils" was performed, what approach would it be coded to? A. external B. open C. via natural or artificial opening D. percutaneous

A. external **Procedures performed with an orifice on structures that are visible without the aid of any instrumentation are coded to the approach external. Procedures performed by cutting through the skin or mucous membrane and any other body layers necessary to expose the site of the procedure are coded to the approach open. Procedures performed by entry, puncture or minor incision, of instrumentation through the skin or mucous membrane and any other body layers necessary to reach the site of the procedure are coded to percutaneous. Procedures performed by entry of instrumentation through a natural or artificial external opening to reach the site of the procedure are coded to the via natural or artificial opening approach.

Clinical manifestations of this disease include polydipsia, polyuria, polyphagia, weight loss, and hyperglycemia. Which of the following tests would be ordered to confirm the disease? A. glucose tolerance test B. fasting blood sugar C. glucagon D. postprandial blood sugar

A. glucose tolerance test **A glucose tolerance test shows how well the body metabolizes sugar from foods and if the patient is at risk for diabetes.

Twelve new cases of a certain disease occurred during the month of August. If 4,000 persons were at risk during August, then the A. incidence was 3 per 1,000 persons. B. incidence was 6 per 1,000 persons. C. prevalence was 6 per 1,000 persons. D. prevalence was 3 per 1,000 persons.

A. incidence was 3 per 1,000 persons. **To determine if this was prevalence or incidence, one must know the definition of each. For prevalence, we use the number of known (new and existing) cases for a period / population for the period x 100. For incidence, we use the newly reported cases of a disease for a given period / population at the midperiod x 100. Here, we have new cases of a reported disease that is not stated what the disease is. So we are using the incidence formula. In doing so, we have 12 new cases / 4,000 in August x 100. This gives us 1,200 / 4,000 = .3 or 3 per 1,000 persons.

All of the following items mean the same thing, EXCEPT A. inpatient census. B. daily inpatient census. C. inpatient service day. D. daily census.

A. inpatient census. **Definitions of terms of inpatient service day, daily census, and daily inpatient census are all the same. This is the definition of the inpatient service day where this is a 24-hour period of the facility's choice when the census is taken of all inpatients. Many use midnight as it is quiet then, but this is not required. This also includes any patients who were admitted and discharged within the same day.

This prospective payment system is for ____________________ and utilizes a Patient Assessment Instrument (PAI) to classify patients into case-mix groups (CMGs). A. inpatient rehabilitation facilities B. skilled nursing facilities C. long-term acute care hospitals D. home health agencies

A. inpatient rehabilitation facilities **Inpatient rehabiliatation facilities focus on improving the patient's functionality.

As director of Health Information Services, you are negotiating a contract to purchase a new computerized dictation system that will be used across three satellite ambulatory clinics. What element is most critical in the contract negotiation to ensure use of the software in multiple environments? A. license B. price and payment terms C. delivery terms D. scope and term of warranties

A. license **The license section of the contract specifies what the health care provider can do with the application.

You want to graph the number of patients admitted to three different medical staff services on each day of the last month. Because you have a large number of observations (one for each day of the month) and you want to be able to compare the observations for each of the three services on one data display, your best choice is a A. line graph. B. table. C. histogram. D. bar chart.

A. line graph. **When data needs to be categorized by time, a line graph is the best tool to use. The x-axis shows the time period and y-axis shows the value of the variables. Line graphs can also be called run charts.

In statistics, the notation "ΣXY" means A. multiplied each pair of the X and Y scores and then summed their values. B. summed all the values of the X variable, all the values of the Y variable, and then multiplied all of these results. C. summed all the values of the X variable and then multiplied this result times the values of the Y variable. D. variation of the X and Y variable is multiplied then squared.

A. multiplied each pair of the X and Y scores and then summed their values. **To answer this question, one must know that the symbol "Σ" means the sum of. Then the pair of variables are multiplied and totaled.

In ICD-10-PCS when the objective of the procedure is to cut off the blood supply to the vessel the root operation would be: A. occlusion. B. restriction. C. dilation. D. bypass.

A. occlusion. **Occlusion is completely closing an orifice or lumen of a tubular body part (i.e. cutting off the blood supply to a vessel). Restriction is partially closing an orifice or lumen of a tubular body part. Dilation is expanding an orifice or the lumen of a tubular body part. Bypass is altering the route of passage of the contents of a tubular body part.

An Advance Beneficiary Notice (ABN) is a document signed by the A. patient indicating whether he/she wants to receive services that Medicare probably will not pay for. B. physician advisor indicating that the patient's stay is denied. C. provider indicating that Medicare will not pay for certain services. D. utilization review coordinator indicating that the patient stay is not medically necessary.

A. patient indicating whether he/she wants to receive services that Medicare probably will not pay for. **It is the document, that requires the patient to acknowledge in writing, that he or she wants to have the noted procedure or service, even though payment will most probably require the patient to pay out-of-pocket for the entire cost.

CMS assigns one _______________ to each APC and each ______________ code. A. payment status indicator, HCPCS/CPT B. MS-DRG, CPT C. payment status indicator, ICD-10-CM and ICD-10-PCS D. CPT code, HCPCS

A. payment status indicator, HCPCS/CPT **The payment status indicators are determined per procedure or service reported with CPT and HCPCS Level II codes.

The muscular contractions that move food through the alimentary canal from the mouth to the anus is referred to as A. peristalsis. B. pleural effusion. C. paralysis. D. peritonitis.

A. peristalsis. **The involuntary constriction and relaxation of the muscles of the intestine or another canal create wavelike movements that push the contents of the canal forward.

Documentation of the care provided at the bedside is known as A. point-of-care systems. B. electronic medical record. C. document management system. D. electronic health record.

A. point-of-care systems. **The point-of-care system allows documentation in the EHR to be done at the patient's bedside. This provides quicker access to the current information as well as improved documentation as the care is provided while the health care professional remembers what was done.

A component of the EHR that reports communicable disease is known as A. population health B. health information exchange C. clinical decision support system D. patient provider portal

A. population health **Population health strives to improve the health of the population through mandatory reporting including that of communicable diseases and monitoring health outcomes.

A coder came across a medical record where a sputum culture indicated bacterial pneumonia but the diagnosis did not indicate the cause of the pneumonia. Documentation in the patient's record failed to meet which one of the following criteria: A. precision. B. clarity. C. completeness. D. consistency.

A. precision. **Precision is when clinical documentation indicates a more specific diagnosis than is documented such as a sputum culture that indicates bacterial pneumonia and diagnosis does not indicate the cause. Completeness is when an abnormal lab test results come back but clinical significance of results is not documented. Clarity is when a medical record contains documentation of signs and symptoms, but a definitive diagnosis is not documented. Consistency is when a discrepancy among two or more treating providers regarding a diagnosis is found in the medical record.

During a risk management strategy session, the HIM director states that policies and procedures must be created to prevent specific opportunities for an adverse outcome, as well as those that will function to diminish any that might not be preventable. This type of strategy is known as a _____________ A. proactive strategy B. prevention quality indicator C. reactive strategy D. cost-benefit analysis

A. proactive strategy **Proactive strategies are used to avoid situations or occurrences that are known to result in adverse outcomes.

Ms. Thomas was a patient at your health care facility. She has been told that there are some records that she cannot have access to. These records are most likely A. psychotherapy notes. B. alcohol and drug records. C. a mental health assessment. D. AIDS records.

A. psychotherapy notes. **Psychotherapy health records are not part of the designated record set and therefore cannot be released to the patient. They are for use by the health care professional only.

Maria Giovanni is in the hospital recovering from colon resection surgery. Based on her symptoms, her doctors are concerned about the possibility that she has developed a pulmonary embolism. Which of the following procedures will provide the definitive diagnosis? A. pulmonary angiography B. chest X-ray C. none of these answers apply D. lung scan

A. pulmonary angiography **Pulmonary angiography is a test to see how blood flows through the lung; angiography is an imaging test that uses X-rays and a special dye to see inside the arteries.

When the third-party payer returns a claim due to missing, inaccurate, or invalid information, this is called a A. rejected claim. B. unprocessed claim. C. denied claim. D. clean claim.

A. rejected claim. **Claims with invalid information are rejected; claims with uncovered services are denied.

The HIM supervisor is evaluating software that would utilize electronic logging and monitoring of requests for copies of patient information and develop an accounting of disclosure. What department function is this most useful for? A. release of information/disclosure management B. transcription C. record location/tracking D. record completion

A. release of information/disclosure management **The release of information/disclosure management software would track authorizations received, authorizations pending, and what was sent to complete the authorizations.

In which of the following circumstances would release of information without the patient's authorization be permissible? A. release to third-party payers B. release to insurance companies C. release to state workers' compensation agencies D. release to an attorney

A. release to third-party payers **A hospital under the Privacy Rule is not required to obtain a patient's authorization before sending records from the patient's hospital stay to a physician who is following up with that patient for treatment purposes; and to an insurance company for payment purposes. The Privacy Rule permits uses and disclosures without an individual's authorization: as required by law; for public health activities; victims of abuse, neglect, or domestic violence; health care oversight activities; judicial and administrative proceedings; law enforcement purposes; decedents; cadaveric organ, eye, or tissue donation; research; threat to health and safety; essential government functions; and workers' compensation.

The name of the root operation in ICD-10-PCS that describes "cutting out or off, without replacement, all of a body part" is A. resection. B. extraction. C. removal. D. excision.

A. resection. **Resection is cutting out or off, without replacement, all of a body part. Removal is taking out or off a device from a body part by cutting or by use of force. Excision is cutting out or off, without replacement, a portion of a body part. Extraction is pulling or stripping out or off all or a portion of a body part by the use of force.

Which two theories of negligence are used to hold health care facilities accountable for the actions of its employees? A. respondent superior and B. corporate negligence B. invasion of privacy and fraud C. assault and battery D. defamation and fraud

A. respondent superior and B. corporate negligence **Respondeat superior is the doctrine that establishes health care organizations as liable to the patients for the actions of its employees. The organization holds itself as responsible for the actions of its employees, provided that these individuals were acting within the scope of their employment or with the organization's direction at the time they conducted the tortious activity in question. Under the doctrine of corporate negligence, the organization holds itself as a provider of health care services and owes its patients the duty to use reasonable care to maintain safe and adequate facilities and equipment; select and retain competent medical professionals; oversee all persons who practice medicine with the hospital; and formulate, adopt, and enforce rules and policies that ensure quality care for all patients.

Commercial insurance plans usually reimburse health care providers under some type of __________ payment system, whereas the federal Medicare program uses some type of _________ payment system. A. retrospective, prospective B.retrospective, concurrent C.prospective, retrospective D. prospective, concurrent

A. retrospective, prospective **Retrospective means past—already done. Prospective is future.

Which of the following tasks is the most appropriate for the Health Information Services director to delegate to the supervisor of Record Processing and Statistics? A. reviewing monthly statistical reports to verify accuracy B. interviewing applicants for the position of Tumor Registrar C. completing a performance rating on the assistant director of the department D. formulating a record retention policy for the entire facility

A. reviewing monthly statistical reports to verify accuracy **Delegation is a skill that matches the right employee with the right task according to authority and responsibility. The only option that matches the authority and responsibility of the supervisor of Record Processing and Statistics is the statistical report.

A distribution is said to be positively skewed when the mean is A. shifted to the right. B. multimodal. C. shifted to the left. D. bimodal.

A. shifted to the right. **The student must consider the skewness of the distribution of data and whether this is positive or negative. If the data is positive, it is skewed to the right or the large cluster of scores is on the left side. If the data is negative, it is skewed to the left or the large cluster of scores is on the right side. This question states the mean is positively skewed, so we know that scores have shifted to the right.

An African American couple is undergoing genetic counseling to determine the likelihood of producing children with a recessively genetic blood condition. The genetic tests reveal that the father carries the trait to produce abnormal hemoglobin, HbS, which causes crystallization in RBCs and deforms their shape when O2 is low. This condition causes painful crises and multiple infarcts and is termed A. sickle cell anemia. B. iron-deficiency anemia. C. thalassemia. D. hemophilia.

A. sickle cell anemia. **Sickle cell disease is a genetic condition where the RBCs are abnormally shaped, like crescents, which prevents them from flowing smoothly through the capillaries decreasing the amount of oxygen delivered to the tissues.

A HIPPS (Health Insurance Prospective Payment System) code is a five-character alphanumeric code. A HIPPS code is used by home health agencies (HHA) and ____. A. skilled nursing facilities (SNFs) and inpatient rehabilitation facilities (IRFs) B. ambulatory surgery centers (ASCs) and skilled nursing facilities (SNFs) C. physical therapy (PT) centers and inpatient rehabilitation facilities (IRFs) D. ambulatory surgery centers (ASCs) and physical therapy (PT) centers

A. skilled nursing facilities (SNFs) and inpatient rehabilitation facilities (IRFs) **Inpatient rehabilitation facilities (IRF) reports the HIPPS (Health Insurance Prospective Payment System) code on the claim. The HIPPS code is a five-digit CMG (case-mix group). Therefore, the HIPPS code for a patient with tier 1 comorbidity and a CMG of 0109 is B0109. Home Health Agencies (HHA) report the HIPPS code on the claim. The HIPPS code is a five-character alphanumeric code. The first character is the letter "H." The second, third, and fourth characters represent the HHRG (home health resource group). The fifth character represents what elements are computed or derived. Therefore, the HIPPS code for the HHRG C0F0S0 would be HAEJ1.

Most of the digestion of food and absorption of nutrients occur in the A. small intestine. B. stomach. C. esophagus. D. ascending colon.

A. small intestine. **The esophagus is the tube to the stomach from the mouth, the small intestine is where the breakdown of the food and nutrient absorption occur, and the colon is where most of the water is extruded from the eaten food.

Which of the following is not one of the five layers of the epidermis? A. stratum fascia B. stratum lucidum C. stratum spinosum D. stratum corneum

A. stratum fascia **This skin is divided into five separate layers. In order from most superficial to deepest, they are the stratum corneum, stratum lucidum, stratum granulosum, stratum spinosum, and the stratum basale.

Which set of muscles controls the movement of the eye up and down? A. superior and inferior rectus B. medial and lateral rectus C. medial and lateral oblique D. superior and inferior oblique

A. superior and inferior rectus **The superior rectus is an extraocular muscle that attaches to the top of the eye and it moves the eye upward. The inferior rectus is an extraocular muscle that attaches to the bottom of the eye.

The quality review process of invasive and noninvasive procedures to ensure performance of appropriate procedure, preparation of patient, monitoring and postoperative care, and education of patient describes A. surgical review. B. infection review. C. blood and blood component usage. D. universal protocol.

A. surgical review. **The terms "invasive and non-invasive" are what point to surgical review.

Which type of joint, such as the sutures of the skull, has no movement? A. synarthrosis B. amphiarthrosis C. arthritis D. diarthrosis

A. synarthrosis **Synarthrosis is an immovably fixed joint between bones connected by fibrous tissue (e.g., the sutures of the skull).

The nose, mouth, sinuses, pharynx, and larynx make up the A. the upper respiratory tract. B. lower gastrointestinal tract. C. the lower respiratory tract. D. upper gastrointestinal tract.

A. the upper respiratory tract. **The major passages and structures of the upper respiratory tract include the nose or nostrils, nasal cavity, mouth, throat (pharynx), and voice box (larynx).

Resources that produce little-to-no value to the organization, such as non-utilized talent, inventory miscalculations, and staff members waiting for information so they can do their jobs, is known as _______, which must be reduced, if not eliminated, in a lean managed organization. A. waste B. benchmarking C. measured improvements D. value

A. waste **Waste is the use of resources that product little-to-no value to the organization, staff, or patients.

Select the information system that is used to increase the quality of documentation and improve reimbursement. A. EHR B. CDI C. Encoder and grouper D. ROI

B. CDI **Gesture recognition technology is the ability of the computer to read handwritten free text, including that in cursive.

Southwest Health System has numerous semiretired staff. The Human Resources Department has provided training regarding the Age Discrimination in Employment Act (ADEA). They emphasized that it protects employees and applicants beginning at what age? A. 50 B. 40 C. 70 D. 60

B. 40 **The Age Discrimination in Employment Act (ADEA) of 1967 protects applicants and workers age 40 and older from job discrimination.

In ICD-10-PCS, there are how many different approaches to reach the site of the procedure? A. 6 B. 7 C. 5 D. 4

B. 7 There are seven different approaches: 1. Open 2. Percutaneous 3. Percutaneous Endoscopic 4. Via Natural or Artificial Opening 5. Via Natural or Artificial Opening Endoscopic 6. Via Natural or Artificial Opening Endoscopic with Percutaneous Endoscopic Assistance 7. Via External

The Level II (national) codes of the HCPCS coding system are maintained by the A. American Medical Association. B. Centers for Medicare and Medicaid Services. C. CPT Editorial Panel. D. local fiscal intermediary.

B. Centers for Medicare and Medicaid Services. **HCPCS was developed by the Centers for Medicare and Medicaid (CMS) for the same reasons that the AMA developed CPT: for reporting medical procedures and services. The Centers for Medicare and Medicaid Services (CMS) updates HCPCS Level II quarterly on January 1, April 1, July 1, and October 1.

I have been asked to select the messaging standard that will be used for our new PACS. My recommendation is: A. United Medical Language System B. Digital Imaging and Communications in Medicine C. MEDCIN D. ANSI Accredited Standards Committee X12N

B. Digital Imaging and Communications in Medicine **Digital Imaging Communications in Medicine (DICOM) is the standard used to share medical images such as those in a PACS.

A patient is being cared for in his or her home by a qualified agency participating in Medicare. The data-entry software used to conduct all patient assessments is known as A. HHRG. B. HAVEN. C. IRVEN. D. RBRVS.

B. HAVEN. **Remember, many consider their own homes their HAVEN.

Miles has asked you to explain the rights he has via HIPAA privacy standards. Which of the following is a right that you would explain? A. He can discuss financial arrangements with business office staff. B. He can ask to be contacted at an alternative site. C. He can review his bill. D. He can ask a patient advocate to sit in on all appointments at the facility.

B. He can ask to be contacted at an alternative site. **One of the patient rights is that they can request the covered entity contact them through an alternative method. This could include post office boxes, their office, and other locations.

The PQRS is a reporting system established by the federal government for physician practices who participate in Medicare for quality measure reporting. Beginning in 2017, this program transitioned into A. NCQA. B. MIPS. C. PQRS. D. OIG.

B. MIPS. **2016 was the last program year for PQRS. PQRS transitioned to the Merit-Based Incentive Payment System (MIPS) under the Quality Payment Program.

Which type of diagnostic test involves the use of electromagnetic waves to produce very detailed images of soft tissue structures of the body? A. CAT or CT scan B. MRI C. X-ray D. electrophoresis

B. MRI **Magnetic resonance imaging (MRI) is a noninvasive medical test used to diagnose medical conditions. It uses a powerful magnetic field, radio frequency pulses, and a computer to produce detailed pictures of organs, soft tissues, bone, and virtually all other internal body structures.

An accreditation agency counterpart to the Joint Commission for managed care organizations is the A. AHRQ. B. NCQA. C. IOM. D. AHCPR.

B. NCQA. **The National Committee for Quality Assurance (NCQA) is an independent nonprofit organization that works to improve health care quality through the administration of evidence-based standards, measures, programs, and accreditation.

When operating under the Health Insurance Portability and Accountability Act of 1996, what is a basic tenet in information security for health care professionals to follow? A. The information system encourages mass copying, printing, and downloading of patient records. B. Security training is provided to all levels of staff. C. Patients are not educated about their right to confidentiality of health information. D. When paper-based records are no longer needed, they are bundled and sent to a recycling center.

B. Security training is provided to all levels of staff. **According to the administrative safeguards, entities must provide security training for all staff.

Which of the following best describes the situation of a provider who agrees to accept assignment for Medicare Part B services? A. The provider is a nonparticipating provider. B. The provider cannot bill the patients for the balance between the MPFS amount and the total charges. C. The provider is paid according to the Medicare Physician Fee Schedule (MPFS) plus 10%. D. The provider is reimbursed at 15% above the allowed charge.

B. The provider cannot bill the patients for the balance between the MPFS amount and the total charges. **Since the provider accepts assignment, he or she will accept the Medicare Physician Fee Schedule (MPFS) payment as payment in full.

You have been asked to establish guidelines on screen design that will be used in all of your information system projects. Which of the following standards would be included in your guidelines? A. Eliminate all hyperlinks. B. The screen should read right to left. C. Provide instructions on how to complete the screen. D. Use red and green to flag key data.

B. The screen should read right to left. **There are best practices for screen design such as screens should have the same look and feel, the fields should be in a logical order, and the screen should be read from left to right.

Critique this statement: A business associate has the right to use a health care facility's information beyond the scope of their agreement with the health care facility. A. This is a false statement because the HIPAA Privacy Rule states that to use it in their own business, they must have the health care facility's approval. B. This is a false statement because it is prohibited by the HIPAA Privacy Rule. C. This is a true statement as long as they have patient consent. D. This is a true statement because business associates can use the information for their main source of business as long as the patient's privacy is protected.

B. This is a false statement because it is prohibited by the HIPAA Privacy Rule. **PHI can only be used by the business associates for the purpose they are hired to perform. This would need to be specified in the business associate agreement.

The nursing initial assessment upon admission documents the presence of a decubitus ulcer. There is no mention of the decubitus ulcer in the physician documentation until several days after admission. The present on admission (POA) indicator is A. N = not present at the time of inpatient admission. B. U = documentation is insufficient to determine if condition was present at the time of admission. C. W = provider is unable to clinically determine if condition was present at the time of admission. D. Y = present at the time of inpatient admission.

B. U = documentation is insufficient to determine if condition was present at the time of admission. **Query the physician as to whether the decubitus ulcer was present on admission or developed after admission.

An example of a valid code in ICD-10-CM is A. BJRT23x. B. Z3A.34. C. 576.212D. D. 329.6677.

B. Z3A.34. **ICD-10-CM codes begin with an alphabetical letter. The second character is always numeric. Characters 3 through 7 can be alpha or numeric. There is a decimal after the third character. Codes can consist of three to seven characters.

Which endocrine gland secretes epinephrine, which activates the "fight or flight" response and increases blood pressure and metabolism? A. alpha cells of the pancreas B. adrenal medulla C. beta cells of the pancreas D. pineal

B. adrenal medulla **The hormones that are secreted by the adrenal medulla are epinephrine—also known as adrenaline—and norepinephrine, which is the hormone that works with epinephrine in responding to physical and emotional stress.

Which of the following issues seems to be the main contributing factor to the rise of antibiotic-resistant bacterial organisms such as methicillin-resistant Staphylococcus aureus? A. patient noncompliance in the use of antibiotics when the patient does not take all of the medicine for the allotted time B. all answers apply C. the use of antibacterial soaps and other products that contain triclosan and/or triclocarban D. the overprescribing of antibiotics to treat bacterial and viral infections

B. all answers apply **All of these are contributing factors to the rise of multidrug resistant organisms, including Tuberculosis and C. difficile. The high percentage of animals raised on antibiotics is also considered by many to be a contributing factor.

Which of the following measures should a health care facility incorporate into its institution-wide security plan to protect the confidentiality of the patient record? A. use of unique computer passwords, key cares, or biometric identification B. all of these answers apply. C. verification of employee identification D. locked access to data processing and record areas

B. all of these answers apply. **Organizations should limit access to the minimum necessary, require use of identification or passwords, implement encryption devices for transmitted data, install protective hardware and software devices including firewalls, eliminate open network jacks in unsecured areas, routinely audit access to patient formation through audit trails, and ensure that all data processing and record areas are secure.

The first character for all of the codes assigned in ICD-10-CM is A. a digit. B. an alphabet. C. an alphabet or a number. D. a number.

B. an alphabet. **The appropriate codes are codes from A00.0 through T88.9, Z00-Z99.8 that must be used to identify diagnoses, symptoms, conditions, problems, complaints, or other reasons for the encounter/visit. The first character of ICD-10-CM is always an alpha character. The second character is always numeric. Characters 3 through 7 can be alpha or numeric.

A severe allergic reaction that can be life-threatening, due to a swollen throat, difficulty breathing, rash, and dizziness that must be treated with injectable epinephrine is termed A. asthma. B. anaphylaxis. C. arrhythmia. D. amyloidosis.

B. anaphylaxis. **Anaphylaxis is a rare, generalized, potentially life-threatening allergic reaction to a particular substance (allergen) to which individuals have previously developed an extreme sensitivity (hypersensitivity).

Under APCs, payment status indicator "X" means A. significant procedure, multiple procedure reduction applies. B. ancillary services. C. significant procedure, not discounted when multiple. D. clinic or emergency department visit (medical visits).

B. ancillary services. **Under the APC system, there exists a list of status indicators (also called service indicators, payment status indicators, or payment indicators). This indicator is provided for every HCPCS/CPT code and identifies how the service or procedure would be paid (if covered) by Medicare for hospital outpatient visits.

In ICD-10-CM, the final character of the code indicates laterality. An unspecified side code is also provided should the site not be identified in the medical record. If no bilateral code is provided and the condition is bilateral, the ICD-10-CM Official Coding Guidelines direct the coder to A. assign the unspecified side code. B. assign separate codes for both the left and right side. C. not assign a code. D. query the physician.

B. assign separate codes for both the left and right side. **ICD-10-CM codes that indicate laterality specifically classify conditions that occur on the left, right, or bilaterally. If a bilateral ICD-10-CM code is not provided and the condition is bilateral, assign separate codes for both the left and right side. If the side is not identified in the patient record, assign a code for unspecified site.

The comparing of one facility's outcomes to another, similar facility, or to national standards is known as ______________ A. value-based B. benchmarking C. control charting D. nominal group technique

B. benchmarking **Benchmarking is the process of comparing of one facility's outcomes to another, similar facility, or to national standards.

THe healthcare facility has decided to purchase all of their information systems from Vendor A so that the information systems will be integrated. This system selection strategy is known as A. interfaced B. best of fit C. best of breed D. go-live

B. best of fit **Information systems created by a vendor are designed to work together (integrated).

Case definition is important for all types of registries. Age will certainly be an important criterion for accessing a case in a(n) _____________ registry. A. trauma B. birth defects C. implant D. HIV/AIDS

B. birth defects **Birth defects registries collect information on newborns with birth defects. The registry contains info on children up to a certain age born with a birth defect.

A disease of the inner ear with fluid disruption in the semicircular canal that causes vertigo is A. mastoiditis. B. both labyrinthitis and Meniere's disease. C. labyrinthitis. D. Meniere's disease.

B. both labyrinthitis and Meniere's disease. **Labyrinthitis and Meniere's disease have similarities but are different disorders. Both are conditions affecting the inner ear. Both can cause vertigo and affect a person's balance. In addition, the two have similar symptoms and risk factors. But labyrinthitis is curable, whereas Meniere's disease is incurable.

A list or collection of clinical words or phrases with their meanings is a A. language. B. clinical vocabulary. C. medical nomenclature. D. data dictionary.

B. clinical vocabulary. **A clinical vocabulary lists words or phrases with their meanings, provides for the proper use of clinical words as names or symbols, and facilitates mapping standardized terms to broader classifications for administrative, regulatory, oversight, and fiscal requirements.

An 81-year-old male with arteriosclerosis and a long-standing history of taking Coumadin presents to his physician's office for his biweekly prothrombin time (PT) test. The PT test is one of the most common hemostasis tests used as a presurgery screening and monitoring Coumadin (warfarin) therapy. This test evaluates A. the type and cross-match of blood. B. coagulation of the blood. C. the oxygen-carrying capacity of RBCs. D. the iron-binding capacity of RBCs.

B. coagulation of the blood. **Prothrombin time (PT) is a blood test that measures how long it takes blood to clot; a PT test can be used to check for bleeding problems. PT is also used to check whether medicine to prevent blood clots is working. A PT test may also be called an INR test.

Willful disregard of a subpoena is considered A. abuse of process. B. contempt of court. C. breach of contract. D. contributory negligence.

B. contempt of court. **Failure to respond to a subpoena may result in a court order compelling attendance or a contempt of court citation.

The causative organism for severe acute respiratory syndrome (SARS) is a A. fungus. B. coronavirus. C. retrovirus. D. bacterium.

B. coronavirus. **A new virus, in the coronavirus family, was discovered to have caused SARS in 2002. A zoonotic virus, it probably originated in civet cats, which are considered a delicacy, in China.

There are times when good staff members can burn out. As a manager, a _________ program can help the organization retain these quality workers while providing the staff member with something new to do. A. rewards B. cross-training C. intellectual capital D. reactive strategy

B. cross-training **Cross-training programs provide education and training in different roles or departments.

After receiving a completed requisition to fill positions within the HIM department, Human Resources can be most effective in recruiting qualified applicants with the assistance of a(n) A. salary schedule. B. current job description. C. employee benefits handbook. D. departmental organizational chart.

B. current job description. **Human Resources personnel should be familiar with the job duties outlined in the job description when recruiting employees. Job descriptions should be reviewed and updated before beginning the hiring process.

When you and your team have identified a problem and must determine the cause or causes of that situation, completing a(n) ________ can help. A. matrix diagram B. fishbone diagram C. flowchart D. histogram

B. fishbone diagram **A fishbone diagram is used to determine the cause or causes of a condition or situation.

What is the name of the protein that gives breads, cereals, baked goods, and pasta their structure but can cause serious gastrointestinal complications when ingested by individuals with celiac disease? A. glucose B. gluten C. carbohydrate D. antioxidant

B. gluten **This glue-like property makes the dough elastic and gives bread the ability to rise when baked and provides a chewy, satisfying texture. Gluten is a family of proteins found in grains like wheat, rye, spelt, and barley. When flour is mixed with water, the gluten proteins form a sticky network that has a glue-like consistency.

A cardiac condition where there is an abnormality in the way the electricity passes through the sinoatrial (SA) node and the atrioventricular (AV) node to generate a pulse or heartbeat is called A. tachycardia. B. heart block. C. hypertrophic cardiomyopathy. D. acute myocardial infarction.

B. heart block. **Heart block is an abnormal heart rhythm where the heart beats too slowly (bradycardia); with this condition, the electrical signals that tell the heart to contract are partially or totally blocked between the upper chambers (atria) and the lower chambers (ventricles).

ICD-10-PCS was implemented in the United States to code A. physician office procedures. B. hospital inpatient procedures. C. hospital inpatient diagnoses. D. hospital outpatient diagnoses.

B. hospital inpatient procedures. **The International Classification of Diseases, 10th Revision, Procedure Coding System (ICD-10-PCS) is an entirely new procedure classification system developed by the Centers for Medicare and Medicaid Services (CMS) for use in the United States for inpatient hospital settings only.

Protected health information includes A. only electronic individually identifiable health information. B. individually identifiable health information in any format stored by a health care provider or business associate. C. individually identifiable health information in any format stored by a health care provider. C. only paper individually identifiable health information.

B. individually identifiable health information in any format stored by a health care provider or business associate. **PHI includes all individually identified health information, regardless of format. ePHI, however, includes only electronic PHI.

The prospective payment system (PPS) requiring the use of DRGs for inpatient care was implemented in 1983. This PPS is used to manage the costs for A. home health care. B. inpatient hospital stays. C. medical homes. D. assisted living facilities.

B. inpatient hospital stays. **The prospective payment system is a payment system used in conjunction with Part A Medicare. Reimbursement is provided according to the insured or patient's condition at the time of admission to the hospital.

Under APCs, payment status indicator "C" means A. significant procedure, multiple procedure reduction applies. B. inpatient procedures/services. C. ancillary services. D. significant procedure, not discounted when multiple.

B. inpatient procedures/services. **Under the APC system, there exists a list of status indicators (also called service indicators, payment status indicators, or payment indicators). This indicator is provided for every HCPCS/CPT code and identifies how the service or procedure would be paid (if covered) by Medicare for hospital outpatient visits.

A fetal death occurring during the 21st week of pregnancy, weighing 1,000 g is considered to be a(n) A. early fetal death. B. intermediate fetal death. C. late fetal death. D. preterm neonate.

B. intermediate fetal death. **To answer this question, one must know the classifications of what the early, intermediate, and late fetal death classifications are by the number of weeks gestation and the weight. An early fetal death is either 500 grams or less and less than 20 weeks gestation. An intermediate fetal death is 501 grams to 1,000 grams and between 20 weeks and 28 weeks gestation. This is the category that fits the scenario. The student must know these categories.

Which of the following is the most effective method for the prevention of influenza? A. FluMist nasal spray vaccine B. intramuscular injection vaccine C. coughing into the crook of the elbow D. handwashing

B. intramuscular injection vaccine **A yearly flu shot is the best method of preventing influenza due to the mutations the organism experiences as it circles the globe.

The facility has used Internet technology in order to establish an area restricted to employees where policies and procedures, forms, and more can be stored. This is a(n) A. health information exchange B. intranet. C. extranet. D. portal.

B. intranet. **The intranet uses intranet technology to restrict access of employees to information such as policies and procedures, forms, cafeteria menu, phone directory, and more.

The primary difference between an experimental (randomized) clinical trial and other observational study designs in epidemiology is that in an experimental trial, the A. study and control maps are selected on the basis of exposure to the suspected causal factor. B. investigator determines who is and who is not exposed. C. study is prospective. D. study is case controlled.

B. investigator determines who is and who is not exposed. **The definition of an experimental study is a data collection technique that answers the question "If this is done under carefully controlled conditions, what will happen?" Experimental research allows the researcher to control the variables in the study. Usually the researcher manipulates one variable and controls or randomizes the rest of the variables. Observational research involves observing the participant(s). This may include group dynamics of listening, observing, and conversing with participants rather than gathering written facts from the participants. The researcher may guide the discussion but does not lead the participants to give certain answers.

The physician performed a fiberoptic bronchoscopy with irrigation of the bronchus. In this scenario, what should the coder code? A. unspecified code B. irrigation of bronchus only C. bronchoscopy only D. bronchoscopy and irrigation of bronchus

B. irrigation of bronchus only **Inspection of a body part that is performed to achieve the objective of a procedure is not coded separately. So in this case only the irrigation of bronchus would be coded, not the bronchoscopy procedure which allows for the inspection of the body part.

During orientation, a new employee receives the employee handbook. All of the following information about the employee handbook is true, EXCEPT that A. it must be reviewed periodically by legal counsel to avoid legal risk. B. it provides a contractual obligation to continued employment. C. a receipt must be documented in writing. D. it provides policies and procedures developed by management.

B. it provides a contractual obligation to continued employment. **The employee handbook contains information about the organization's policies, procedures, and so forth. It is not a contractual arrangement.

An area identified for needed improvement through benchmarking and continuous quality improvement is known as a A. measure hierarchy. B. key performance indicator. C. knowledge base. D. key attribute.

B. key performance indicator. **Key performance indicators are defined as metrics used to measure key business processes and reflect strategic performance throughout the organization.

The least invasive restrictive gastric surgery used to reduce the size of the stomach to facilitate weight loss in obese patients is A. a gastric bypass. The least invasive restrictive gastric surgery used to reduce the size of the stomach to facilitate weight loss in obese patients is A. a gastric bypass. B. laparoscopic gastric banding. C. a biliopancreatic diversion. D. Roux-en-Y gastric surgery.

B. laparoscopic gastric banding. **A band around the upper part of the stomach to create a small pouch to hold food, the band limits the amount of food you can eat by making you feel full after eating small amounts of food. After surgery, the physician can adjust the band to make food pass more slowly or quickly through the stomach.

The _____ is the largest solid organ of the body, but the ____ is the largest organ overall. A. skin, brain B. liver, skin C. brain, latissimus dorsi D. intestine, liver

B. liver, skin **The liver is the largest internal solid organ, weighing approximately 3.5 pounds, and the skin is the largest organ (external) overall, with the size dependent upon the height and weight of the individual.

Which one of the following is NOT a pathophysiological factor in anemia? A. excessive RBC breakdown B. loss of spleen function C. lack of RBC maturation D. loss of bone marrow function

B. loss of spleen function **The spleen does not have any impact on the production of red blood cells.

Several recent studies suggest that this issue/condition is now the third leading cause of death in the United States, is responsible for at least 10% of all deaths in the United States, and is an underrecognized and therefore underreported cause of death in the United States. This issue/condition is A. handguns. B. medical errors. C. epidemics. D. car accidents.

B. medical errors. **Medical errors may be the third leading cause of death in the United States, a new study contends. Johns Hopkins University researchers analyzed eight years of U.S. data and concluded that more than 250,000 people died each year due to medical errors.

According to CPT, antepartum care includes all of the following EXCEPT A. physical examination. B. monthly visits up to 36 weeks. C. initial and subsequent history. D. routine chemical urinalysis

B. monthly visits up to 36 weeks. **Antepartum care begins with conception and ends with delivery, and it includes the following: -initial prenatal history and examination -subsequent prenatal history and examinations -documentation of weight, blood pressures, and fetal heart tones -routine chemical urinalyses -monthly visits up to 28 weeks gestation -biweekly visits to 36 weeks gestation -weekly visits until delivery

Which of the following autoimmune diseases affects tissues of the nervous system? A. rheumatoid arthritis B. myasthenia gravis C. Goodpasture's syndrome D. Hashimoto's disease

B. myasthenia gravis **Myasthenia gravis results from an abnormal immune reaction in which antibodies inappropriately attack and gradually destroy certain receptors in muscles that receive nerve impulses (antibody-mediated autoimmune response).

APCs are groups of services that the OPPS will reimburse. Which one of the following services is not included in APCs? A. radiation therapy B. organ transplantation C. screening exams D. preventive services

B. organ transplantation **APC = Ambulatory Payment Classification. OPPS = outpatient prospective payment services. Organ transplants are not provided in outpatient facilities.

Coinsurance payments are paid by the _______ and determined by a specified percentage. A. facility B. patient (insured) C. physician D. third-party payer

B. patient (insured) **The patient is required to pay, in part, based on a percentage, of the entire cost of the procedure or service.

What is the large serous membrane that covers the abdominal organs and lines the abdominal wall? A. pericardial membrane B. peritoneum C. perineum D. pleural membrane

B. peritoneum **The peritoneum is a continuous transparent membrane that lines the abdominal cavity and covers the abdominal organs also known as viscera.

A student has asked for an example of disclosure of PHI that is allowed by HIPAA. An appropriate example is A. letting a business associate use PHI in whatever manner they see fit. B. permitting a spouse to pick up medication for the patient. C. releasing patient information to the patient's attorney without an authorization. D. mandating that a health care facility has to amend the health record of a patient at the patient's request.

B. permitting a spouse to pick up medication for the patient. **Permitting a spouse to pick up medication at a pharmacy is allowed under HIPAA.

You need to retrieve information on a particular physician in your facility. Specifically, you need to know how many cases he saw during the month of May. What would be your best source of information? A. MEDLINE database B. physician index C. National Practitioner Data Bank (NPDB) D. Healthcare Integrity and Protection Data Bank (HIPDB)

B. physician index **The physician index is a listing of cases in order by physician name or physician identification number. It also includes the patient's health record number and may include other information, such as date of discharge. The physician index enables users to retrieve information about a particular physician, including the number of cases seen during a particular time period.

Under the RBRVS, each HCPCS/CPT code contains three components, each having assigned relative value units. These three components are A. fee-for-service, per diem payment, and capitation. B. physician work, practice expense, and malpractice insurance expense. C. geographic index, wage index, and cost of living index. D. conversion factor, CMS weight, and hospital-specific rate.

B. physician work, practice expense, and malpractice insurance expense. **Medicare uses three componets of each procedure: physician work, practice expense, and malpractice insurance portion; to calculate the RVU for each code.

The HITECH breach notification requirement is significant and different from any other requirements in the past because it A. requires the entity responsible to pay monetary damages to those affected by the breach. B. places organizations on the radar of regulatory agencies and in the media spotlight. C. suspends the responsible entity from being able to disclose PHI for a specific period of time. D. requires the CEO of the entity responsible to serve jail time.

B. places organizations on the radar of regulatory agencies and in the media spotlight. **HITECH added breach notification requirements for entities with custody of patient information. HITECH places organizations on the radar of regulatory agencies and in the media spotlight when PHI is handled inappropriately. It also extends consequences to entities not previously bound by HIPAA.

Traditional management functions, such as ________, must be applied to performance improvement initiatives. A. reimbursement B. planning C. accreditation D. education

B. planning **Proper planning contributes heavily to productivity and efficiency.

Which one of the following is NOT a data retrieval tool? A. sound B. point-and-click fields C. icons D. color

B. point-and-click fields **A point-and-click field is not a data retrieval tool.

A barrier to widespread use of automated code assignment is A. inadequate technology. B. poor quality of documentation. C. resistance by physicians. D. resistance by HIM professionals.

B. poor quality of documentation. **Computer-assisted coding (CAC) is a tool intended for improved efficiency of the coding and claims submission process. While CAC can be useful in settings where documentation is structured and has a limited vocabulary, it is critical to remember that accurate documentation is the underlying resource for technological advances relating to patient records, medical coding, and quality improvement. If the medical record does not contain reliable and valid documentation, it cannot be coded correctly by an automated system.

Installation of a new document management system is scheduled for the HIM department. The department director calls a meeting of the HIM supervisors to coordinate plans for training the HIM staff. What key factors must be taken into consideration as part of the training preparation process? A. the total storage capacity and price of the new system B. presentation, content, and scheduling to accommodate all employee shifts C. collecting the results of the RFP responses and vendor selection process used during the selection process D. scheduling training times and locations for all physicians and hospital personnel who will use the system

B. presentation, content, and scheduling to accommodate all employee shifts **The supervisors must focus on the method of presentation, the content of the training, and scheduling the sessions to accommodate all employee shifts.

The method of calculating errors in a coding audit that allows for benchmarking with other hospitals, and permits the reviewer to track errors by case type, is the A. benchmarking method. B. record-over-record method. C. focused review method. D. code method.

B. record-over-record method. **The record-over-record method divides the number of records correctly coded by the total number of records in the sample. It is less labor intensive, widely recognized, and focused on statistics. However, it is more subjective in that it does not have a definition of what counts as an error and educational opportunities are not easily identified.

The major purpose of random assignment in a clinical trial is to A. ensure that the study groups are comparable on baseline characteristics. B. reduce selection bias in allocation of treatment. C. facilitate double-blinding. D. help ensure that study subjects are representative of the general population.

B. reduce selection bias in allocation of treatment. **A clinical trial uses random assignment because it must have as little selection bias as possible in the treatment allocated.

This is the amount collected by the facility for the services it bills. A.charges B. reimbursement C. costs D. contractual allowance

B. reimbursement **The term reimbursement means compensation for medical services.

Which of the following is usually the largest expense for a health care facility? A. release of Information fees B. staffing expenses C. travel fees D. paper supply

B. staffing expenses **Staffing expenses usually consume a major portion of any health care organization's budget.

A major disadvantage of cross-sectional studies is that A. they are usually more expensive and can take a long time to complete. B. the time sequence of exposure and disease is usually not known. C. they cannot provide information on both exposure and disease status in the same individual. D. prevalence rates cannot be calculated.

B. the time sequence of exposure and disease is usually not known. **Disadvantage of cross-sectional studies = Time sequence of exposure and disease is usually not known.

The code structure for ICD-10-CM differs from the code structure of ICD-9-CM. An ICD-10-CM code consists of A. 10 characters. B. three to seven characters. C. seven digits. D. five alphanumeric characters.

B. three to seven characters. **ICD-10-CM codes are composed of codes with 3, 4, 5, 6, or 7 characters. ICD-9-CM codes are composed of codes with 3, 4, or 5 characters.

In order to be secure, data has to be A. unreadable. B. unreadable, unusable, and indecipherable. C. indecipherable. D. unusable.

B. unreadable, unusable, and indecipherable. **Secure information is defined as unreadable, unusable, and indecipherable.

To accomplish the Joint Commission's safety goal to eliminate wrong-site, wrong-patient procedures, the organization can use all of these EXCEPT A. preoperative verification processes. B. using imaging guidance on all procedures. C. available patient records. D. mark the surgical site.

B. using imaging guidance on all procedures. **Imaging guidance will not impact errors such as wrong side or wrong patient.

The main difference between concurrent and retrospective coding is A. what classification system is used. B. when the coding is done. C. the involvement of the physician. D. the credentials of the coder.

B. when the coding is done. **Concurrent coding is completed while the patient is still admitted to the facility whereas retrospective coding is completed after the patient is discharged from the facility.

You have been asked to identify and display the existing record processing workflow with the paper record. After you finish this, you will redesign the process and display the new process. What tool should you use? A. data dictionary B. workflow diagram C. data mapping D. policy and procedure

B. workflow diagram **The workflow diagram shows the steps in the process. In this case, it would be developed to show the current process and then the diagram would be revised to show the new process.

In explaining the benefits of a document management system, you include: A. free text. B. workflow. C. structured data entry. D. natural language processing

B. workflow. **A document management system is when a piece of paper, such as a page from the paper health record, is scanned. The document is stored as an image in the database. Workflow technology is used to allow coders, analysts, and others access to the health record and to perform their activity when the health record is available.

The police came to the HIM department today and asked that a patient's right to an accounting of disclosure be suspended for two months. What is the proper response to this request? A. "I'm sorry, Officer, but privacy regulations do not allow us to do this." B. "Certainly, Officer. We will take care of that right now." C. "Certainly, Officer. We will be glad to do that as soon as we have the request in writing." D. "I'm sorry, Officer, but we can only do this for one month."

C. "Certainly, Officer. We will be glad to do that as soon as we have the request in writing." **HIPAA allows law enforcement to suspend a patient's ability to obtain an accounting of disclosure so he or she will not learn law enforcement accessed his or her health record. The request must be in writing.

Some services are performed by a nonphysician practitioner (such as a physician assistant). These services are an integral yet incidental component of a physician's treatment. A physician must have personally performed an initial visit and must remain actively involved in the continuing care. Medicare requires direct supervision for these services to be billed. This is called A. "Assignment" billing. B. "Technical component" billing. C. "Incident to" billing. D. "Assistant" billing.

C. "Incident to" billing. **Incident to—indicating that this was provided by a nonphysician professional under the supervision of a physician.

Sandy Beach Hospital reports 1,652 discharges for September. The infection control report documents 21 hospital-acquired infections and 27 community-acquired infections for the same month. What is the community-acquired infection rate? A. 2.9 B. 1.4 C. 1.6 D. 1.3

C. 1.6 **The number of community acquired infections (27) divided by the total discharges for the month (1,652) and then multiplied by 100 to get the percent = 1.6% OR (27 x 100) / 1,652 = 1.6%

ICD-10-PCS codes have a unique structure. An example of a valid code in the ICD-10-PCS system is A. L03.311. B. 013.2 C. 2W3FX1Z. D. B2151.

C. 2W3FX1Z. **ICD-10-PCS codes contain 7 characters that can be letters or numbers. Each code is made up of any one of the 10 digits 1-9 and any one of 24 letters A-H, J-N, and P-Z. The letters "O" and "I" are not used so as not to be confused with the numbers "0" and "1."

Historically, individually identifiable information for deceased individuals regardless of the time of death was covered under the HIPAA Privacy Rule. However, under the HITECH, individually identifiable information for deceased individuals is no longer covered after _____ years. A. 30 B. 40 C. 50 D. 60

C. 50 **Under the HITECH, individually identifiable heath information of persons deceased more than 50 years is no longer protected by the Privacy Rule.

You have been conducting productivity studies on your coders and find that 20% of their time is devoted to querying physicians about missing or unclear diagnoses. Assuming your coders work a 7-hour day, how many minutes do they spend per day querying physicians? A. 56 B. 21 C. 84 D. 140

C. 84 **To calculate productivity on this question, begin by calculating how many minutes they work in a day and then calculate what 20% of that time is. They work 7 hours per day. There are 60 minutes per hour. So to calculate the minutes for one day, multiply 7 x 60 = 420 minutes per day. Now, calculate 20% of 420 minutes by multiplying .20 x 420 = 84. So 84 minutes per day is spent querying physicians.

The Level I (CPT) codes of the HCPCS coding system are maintained by the A. American Hospital Association. B. local fiscal intermediary. C. American Medical Association. D. Centers for Medicare and Medicaid Services.

C. American Medical Association. **HCPCS Level I includes the five-digit (and some five-character) CPT codes developed and published by the American Medical Association (AMA). The AMA is responsible for the annual update of this coding system and its two-digit modifiers.

A patient was denied access to his PHI. He asked for an appeal of the decision and was allowed the appeal. Which circumstance might explain why? A. The CE is exempt from CLIA. B. Patient is part of research and has agreed to a temporary suspension of his rights. C. Another person may be harmed by the release. D. Patient is an inmate and release may cause safety concern.

C. Another person may be harmed by the release. **In some circumstances, the patient does not have the right to appeal a decision to deny access to his PHI. The patient can have an appeal if the physician believes that releasing the PHI would endanger another person.

Which of the following agencies is empowered to implement the law governing Medicare and Medicaid? A. Joint Commission B. Institutes of Health C. Centers for Medicare and Medicaid Services (CMS) D. Department of Health and Human Services

C. Centers for Medicare and Medicaid Services (CMS) **The Centers for Medicare and Medicaid Services (CMS) is an agency of the Department of Health and Human Services (HHS) that develops compliance program guidance specific to Medicare fee-for-service contractors to promote adherence to all Medicare statutory and regulatory requirements. It is also responsible for parts of Medicaid.

In which type of facility does the Privacy Act of 1974 permit patients to request amendments to their medical record? A. mental health and chemical dependency facility B. university-based teaching facility C. Department of Defense health care facility D. private proprietary health care facility

C. Department of Defense health care facility **The Privacy Act of 1974 grants individuals the right to learn what information has been collected about them by federal agencies and their contractors. It also allows individuals to obtain a copy of the information and maintain limited control over the disclosure of that information to other persons or entities. This act is limited to federal health care organizations such as the Veterans Health Administration (VHA), the Indian Health Service, and their contractors.

What advice should be given to a physician who has informed you that she recently discovered that a significant portion of a discharge summary she dictated last month was left out? A. Redictate the discharge summary and replace the old one with the new one. B. Squeeze in the information omitted by writing in available spaces such as the top, bottom, and side margins. C. Dictate the portion omitted with the heading "Discharge Summary—Addendum" and make a reference to the addendum with a note that is dated and signed on the initial Discharge Summary (e.g., "9/1/17-See Addendum to Discharge Summary"—Signature). D. Inform the physician that nothing can be done about the situation.

C. Dictate the portion omitted with the heading "Discharge Summary—Addendum" and make a reference to the addendum with a note that is dated and signed on the initial Discharge Summary (e.g., "9/1/17-See Addendum to Discharge Summary"—Signature). **An amendment (addendum) is a type of late entry in which information is added to support or clarify a previous entry. The date and time of the addendum should be documented; write "addendum" and state the reason for the addendum, referring back to the original or parent document or entry. An addendum should be completed as soon as possible after the original entry. If it is an electronic record, the addendum should be linked to the original entry in the EHR.

You have been asked how to handle versioning of documents in the document management system. Which of the following should be your response? A. Do not allow edits to the information system. B. Display both versions side by side. C. Display the second version, but identify that a previous version exists. D. Delete the first version and retain the second.

C. Display the second version, but identify that a previous version exists. **Versioning is controlling which document can be accessed when the document has been changed—the original or the revised. The original version should still be accessible as this is a legal document.

The legislation that required all federally funded facilities to inform patients of their rights under state law to accept or refuse medical treatment is known as A. living wills. B. durable power of attorney. C. Patient Self-Determination Act. D. advance directives.

C. Patient Self-Determination Act. **The Patient Self-Determination Act requires health care facilities to provide written information on the patient's right to issue advanced directives and to accept or refuse medical treatment.

HIPAA states that release to a coroner is allowed. State law says that the coroner must provide a subpoena. Identify the true statement regarding this situation. A. Follow the HIPAA requirement since it is a federal law. B. You can follow either the state law or the HIPAA rule. C. Follow the state law since it is stricter. D. You must request a ruling from a judge.

C. Follow the state law since it is stricter. **When there is a difference between HIPAA and state law, the stricter law/regulation should be followed. This is known as preemption.

____ is knowingly making false statements or representation of material facts to obtain a benefit or payment for which no entitlement would otherwise exist. A. Whistle-blowing B. Assault C. Fraud D. Abuse

C. Fraud **The act of submitting a request for reimbursement that one is not entitled to is the definition of fraud.

Mappings between ICD-9-CM and ICD-10-CM were developed and released by the National Center for Health Statistics (NCHS) to facilitate the transition from one code set to another. They are called A. Medical Mappings. B. Code Maps. C. GEMS (General Equivalency Mappings). D. ICD Code Maps.

C. GEMS (General Equivalency Mappings). **The National Center for Health Statistics (NCHS) and the CMS annually publish general equivalence mappings (GEMs) that are translation dictionaries or crosswalks of codes that can be used to roughly identify ICD-10-CM codes for their ICD-9-CM equivalent. GEMs facilitate the location of corresponding diagnosis codes between two code sets.

______ requires present on admission (POA) indicator to be assigned to the ___________ diagnosis(es) for all claims for __________________admissions. A. All, principal, inpatient B. All, principal and secondary, inpatient and outpatient C. Medicare, principal and secondary, inpatient D. Medicare, principal, outpatient

C. Medicare, principal and secondary, inpatient **Medicare requires present on admission (POA) indicator to be assigned to the principal and secondary diagnosis(es) for all claims for inpatient admissions.

What Web 2.0 technology would you recommend for someone who wants to share their battle with cancer? A. an extranet B. web pages C. a blog D. a chat room

C. a blog **A blog is a method of sharing your story or other information with others. It can be used to talk about your struggles with cancer, favorite quotes, websites, and much more.

The Cancer Committee at your hospital requests a list of all patients entered into your cancer registry in the last year. This information would be obtained by checking the A. disease index B. tickler file C. accession register D. suspense file

C. accession register **When a case is first entered in the registry, an accession number is assigned; this number is used to identify the patient. The accession number consists of the first digits of the year the patient was first seen at the facility, with the remaining digits assigned sequentially throughout the year. The first case in 2017, for example, might be 17-0001. The accession number may be assigned manually or by the automated cancer database used by the organization. An accession registry of all cases can be kept manually or provided as a report by the database software. This listing of patients in accession number order provides a way to monitor that all cases have been entered into the registry.

Maria has received a request to update a patient's address. What system should she use? A. laboratory information system B. executive information system C. admission-discharge-transfer system D. clinical decision support system

C. admission-discharge-transfer system **The ADT system contains all of the patients who have been seen at the facility and their demographic information including the patient's address.

Of the following, which is a hospital-acquired condition (HAC)? A. traumatic wound infection B. stage I pressure ulcer C. air embolism D. breach birth

C. air embolism **With proper training of health care professionals, an air embolism should not occur.

Which of the following is considered confidential information if the patient is seeking treatment in a substance abuse facility? A. patient's diagnosis B. patient's name C. all of these answers apply D. patient's prognosis

C. all of these answers apply **Special access and disclosure procedures, specifically as it relates to privacy and security, must be followed when handling health records that contain the identity, diagnosis, prognosis, and treatment of patients having a primary or secondary diagnosis of alcohol or drug abuse or, within the patient records of federally assisted programs, any mention of alcohol or drug abuse.

Who of the following who are permitted to have access to patient health records and information without the patient authorization as part of their training program? A. HIM students B. nursing students C. all of these answers apply D. medical students

C. all of these answers apply **Students in training are permitted to have access to patient information without the patient's authorization as long as a contract or letter of agreement between the educational institution and health care provider, or CE, outlines students' responsibilities when accessing PHI, and patient privacy compliance should be maintained by both parties.

Which of the following is a risk factor involved in the etiology of gallstones? A. low-fat diets B. being an adolescent C. being overweight D. the presence of a peptic ulcer

C. being overweight **Gallbladder patients typically have the "4 F's"—fat, female, forty or older, and fertile (premenopause).

Select the data elements that is a foreign key in the patient table. A. last name B. address C. billing number D. health record number

C. billing number **A foreign key is a field in a database that is a primary key in another table. In this case, the foreign key cannot be health record number as that would be the primary key for the table. The only other unique identifier is billing number, which would uniquely identify the patient visit.

The Joint Commission on-site survey process incorporates tracer methodology, which emphasizes surveyor review by means of A. patient tracers. B. policy and procedure manual reviews. C. both system tracers and patient tracers. D. system tracers.

C. both system tracers and patient tracers. **The Joint Commission is responsible for both system quality and patient care quality.

Cancer derived from epithelial tissue is classified as a(n) A. lipoma. B. sarcoma. C. carcinoma. D. adenoma.

C. carcinoma. **Carcinomas, the most commonly diagnosed cancers, originate in the skin, lungs, breasts, pancreas, and other organs and glands.

Henrietta Dawson presents with a chief complaint of pain and weakness in her arms and neck. After an H and P and a review of diagnostic tests that include a myelogram, her doctor diagnoses a herniated nucleus pulposus at the _________ level of her spine. A. lumbar B. sacral C. cervical D. thoracic

C. cervical **Cervical vertebrae are located in the neck. Disc issues in this location would affect the upper limbs.

Changes in case-mix index (CMI) may be attributed to all of the following factors EXCEPT A. changes in support staff composition. B. changes in medical staff composition. C. changes in coding productivity. D. changes in services offered.

C. changes in coding productivity. **Coding productivity will not directly affect CMI. Inaccuracy or poor coding quality can affect CMI. productivity.

The patient has been on medication X for several years. A physician ordered medication Y. The information system immediately sends the physician a message that says that medication Y is contraindicated due to medication X. What technology is being used? A. point-of-care system B. order entry results reporting C. clinical decision support systems D. EHR

C. clinical decision support systems **The clinical decision support feature of the CPOE can look at the patient's medication and allergies to ensure that a medication is contraindicated. If one is, the physician is provided a message so that the medication can be changed or deleted.

When a decision is made to restrict or deny clinical privileges during the recredentialing or reappointment process to a medical staff member, _______________ must be offered. A. privilege suspension B. crisis intervention C. due process D. revocation of license

C. due process **The candidate must be provided with complete and accurate explanation for the restriction or denial to ensure fairness.

A health care organization's compliance plans should not only focus on regulatory compliance, but also have a A. strong personnel component that reduces the rapid turnover of nursing personnel. B. substantial program that increases the availability of clinical data. C. coding compliance program that prevents fraudulent coding and billing. D. component that increases the security of medical records.

C. coding compliance program that prevents fraudulent coding and billing. **Compliance means adhering to rules, laws, standards, and regulations. Coding rules, guidelines, standards, and expected practices must be followed. Coded data is used for multiple purposes and is the basis for which payment and reimbursement decisions are made.

The coding supervisor notices that the coders are routinely failing to code all possible diagnoses and procedures for a patient encounter. This indicates to the supervisor that there is a problem with A. reliability. B. timeliness. C. completeness. D. validity.

C. completeness. **It is very important to have experienced, well-trained coders in order to ensure accurate and complete code assignment and optimum reimbursement. Coders must be skilled at reading through documentation on a variety of different forms and formats and interpreting that documentation to arrive at the correct code assignment.

When a health information professional (record custodian) brings the medical record to court in response to a subpoena duces tecum, it is his or her responsibility to A. explain details of the medical treatment given to the patient. B. leave the original record in the possession of the plaintiff's attorney. C. confirm whether or not the record is complete, accurate, and made in the ordinary course of business. D. present the case favorably for the patient involved.

C. confirm whether or not the record is complete, accurate, and made in the ordinary course of business. **The HIM professional must attest that the record was made in the normal course of business. The health professional confirms the authenticity, correctness, accuracy, and completeness of the record.

This is the difference between what is charged and what is paid. A. customary B. costs C. contractual allowance D. reimbursement

C. contractual allowance **This is the pre-agreed upon maximum amount of reimbursement for each procedure, service, and treatment.

The board of directors of a 400-bed women's hospital receives a report of key quality indicator results on a periodic basis. The report always includes the quarterly cesarean section rate. This reporting period, they see a rise in the rate and want to know if it is a significant increase. What is the best QI tool for this purpose? A. run chart B. scatter diagram C. control chart D. Pareto chart

C. control chart **A control chart is used to study variations in performance over time, and its source.

There are seven criteria for high-quality clinical documentation. All of these elements are included EXCEPT A. consistent. B. precise. C. covered (by third-party payer). D. complete.

C. covered (by third-party payer). **Being covered by a third-party payer has nothing to do with quality clinical documentation.

The supervisor of a Health Information Management department has aspirations of becoming the HIM director. She works very long hours so that she can address all department issues herself rather than relying on her staff to assist her. Which type of management skills is she in need of improving in order to attain her goal of department director? A. political skills B. leadership skills C. delegating skills D. motivating skills

C. delegating skills **Use of delegation expands the manager's capacity, improves the timeliness of decisions, and develops the competencies of other staff members.

Under the inpatient prospective payment system (IPPS), there is a 3-day payment window (formerly referred to as the 72-hour rule). This rule requires that outpatient preadmission services that are provided by a hospital up to three calendar days prior to a patient's inpatient admission be covered by the IPPS MS-DRG payment for A. diagnostic services. B. therapeutic (or nondiagnostic) services whereby the inpatient principal diagnosis code (ICD-10-CM) exactly matches the code used for preadmission services. C. diagnostic and therapeutic services whereby the inpatient principal diagnosis code (ICD-10-CM) exactly matches the code used for preadmission services. D. therapeutic (or nondiagnostic) services whereby the inpatient principal diagnosis code (ICD-10-CM) does not match the code used for preadmission services.

C. diagnostic and therapeutic services whereby the inpatient principal diagnosis code (ICD-10-CM) exactly matches the code used for preadmission services. **The consistency of the diagnosis supports the requirement for continuity of care for the same medical reasons.

There are some circumstances when patients have the right to agree or object to disclosure of protected health information. This includes A. disclosing information to patient's or covered entities' minister. B. disclosing information to patient's attorney. C. disclosing information to a family member who is directly involved in care. D. disclosing information to a family member who is not directly involved in the care.

C. disclosing information to a family member who is directly involved in care. **There are two circumstances when patients have the right to agree or object to disclosure of protected health information. This includes facility directory and disclosing information to family member who is directly involved in care.

Cervical cerclage is a procedure used to help prevent A. breathing restrictions. B. torticollis. C. miscarriage. D. torsion.

C. miscarriage. **Cervical cerclage is the placement of stitches in the cervix to hold it closed; it is used to keep an incompetent cervix from opening early, thereby preventing preterm labor and delivery.

The following services are excluded under the Hospital Outpatient Prospective Payment System (OPPS) Ambulatory Payment Classification (APC) methodology. A. surgical procedures B. radiology/radiation therapy C. durable medical equipment D. clinic/emergency visits

C. durable medical equipment **Durable medical equipment (DME) is only billed when provided for use by the patient in his or her own home.

Which step of the "chain of infection" is disrupted by eliminating the standing water in old tires, birdbaths, and empty buckets when combatting the Zika virus epidemic? A. stopping the direct transmission B. identifying the susceptible host C. eliminating the reservoir D. killing the virus particle

C. eliminating the reservoir **The reservoir is where the organism or its vector resides—eliminate it and there is no place for it to live and breed.

The category "Commercial payers" includes private health insurance companies and A. Medicare/Medicaid. B. Blue Cross Blue Shield. C. employer-based group health insurers. D. TriCare.

C. employer-based group health insurers. **Employer-based group health insurers are commercial and not government-run agencies.

In ICD-10-PCS, to code "removal of a thumbnail," the root operation would be A. removal B. extirpation C. extraction D. fragmentation

C. extraction **Extraction is the pulling or stripping out or off all or a portion of a body part by the use of force. Fragmentation is breaking solid matter in a body part into pieces. Removal is taking out or off a device from a body part by cutting or by use of force. Extirpation is taking or cutting out solid matter from a body part.

Sally wants to motivate her staff members to earn and submit their required continuing education credits by the deadline, so she is offering a $25 gift card to the first one to complete their requirements. Sally is using _____ motivation techniques. A. cross training B. intrinsic C. extrinsic D. intellectual capital

C. extrinsic **Extrinsic motivators are those that offer a reward or acknowledgement from an outside force.

An example of a primary data source for health care statistics is the A. accession register. B. disease index. C. health record. D. MPI.

C. health record. **"Disease index," "accession register," and "MPI" are examples of secondary data sources. The health record is a primary source of data.

The surgeon comes out to speak to a patient's family. He tells them that the patient came through the surgery fine. The mass was benign and they could see the patient in an hour. He talks low so that the other people in the waiting room will not hear, but someone walked by and heard. This is called a(n) A. privacy incident. B. violation of policy. C. incidental disclosure. D. privacy breach.

C. incidental disclosure. **Incidental disclosure is not considered a privacy incident, a violation of policy, or a privacy breach.

Penicillin is effective in the treatment of all of the following diseases EXCEPT A. strep throat. B. syphilis. C. influenza. D. Lyme disease.

C. influenza. **Antibiotics have no effect on viral conditions.

What type of testimony is inappropriate for a health information manager serving as custodian of the record when he or she is called to be a witness in court? A. title and position held in the health care facility B. whether the record is in the practitioner's possession C. interpretation of documentation in the record D. whether the medical record was made in the usual course of business

C. interpretation of documentation in the record **The health information management professional (HIM) is responsible for attesting to the fact of whether or not the record was generated during the usual course of business; the accessibility of the record; the authenticity of the record, whether or not the record was altered; and was the security of the record upheld. The HIM professional is not a clinician and is not responsible for providing care to the patient. The HIM professional is not authorized or clinically trained to interpret documentation written by physicians and other clinicians.

An improper disclosure of patient information to unauthorized individuals, agencies, or news media may be considered a(n) A. defamation. B. libel. C. invasion of privacy. D. slander.

C. invasion of privacy. **The invasion of privacy is the intrusion upon one's solitude. Individuals have the right to be "left alone." This includes the right to be free from surveillance and interference, as well as the right to keep one's information from being disclosed.

A preliminary step prior to issuing a formal RFP, which allows the facility to narrow down the field of potential vendors for procurement of a new hospital-wide computer system, would be to A. initiate contract negotiations. B. create a Gantt chart to monitor project progress. C. issue a request for information (RFI) to potential vendors. D. select the system to be purchased.

C. issue a request for information (RFI) to potential vendors. **The RFI is a document that requests basic information from a vendor. It can be used to determine which vendors you will send the RFP to.

In which gender is the urethra significantly longer? A. it is the same length regardless of gender B. it depends on the height of the individual C. males D. females

C. males **The urethra also allows the semen to be transported through the penis and is therefore longer.

A patient satisfaction survey conducted after discharge is a method of quality measurement through A. prospective indicator. B. process indicator. C. outcomes indicator. D. structure indicator.

C. outcomes indicator. **Patient satisfaction, or lack thereof, is a direct outcome of the patient's stay.

Which of the following is an example of two-factor authentication? A. fingerprint and retinal scan B. username and password C. password and token D. token and smart card

C. password and token **There are three types of access control: what you know, what you have, and what you are. A two-factor authentication would require one method from two of these types of access control such as a password (something you know) and token (something you have).

The patient's blood pressure is automatically being recorded. This is an example of what type of clinical information system? A. medical documentation B. patient registration C. patient monitoring system D. nursing applications

C. patient monitoring system **Patient monitoring systems track vital signs such as the blood pressure and pulse.

The presence of fluid in the alveoli of the lungs is characteristic of A. tuberculosis. B. Crohn's disease. C. pneumonia. D. COPD.

C. pneumonia. **Fluid can collect in the lungs from a variety or organisms or other conditions that trigger the inflammatory response; it is a typical characteristic of pneumonia.

A survey of the patients at your healthcare facility has found that they want access to test results and the ability to schedule appointments online. Because of this, we have decided to implement a(n): A. personal health record B. intranet C. portal D. single sign-on

C. portal **A patient portal allows a patient to view test results, communicate with a provider, schedule appointments, and more.

Under which category of law would Marleana Harrison bring a cause of action against Dr. Billy Ray for disclosing information regarding her previous physical examination to his wife, Jana Ray, who is Ms. Harrison's hairstylist? A. criminal law B. administrative law C. private law D. procedural law

C. private law **Private law is the collective rules and principles that define the rights and duties of people and private businesses. It consists of actions by one or more individuals acting in a private capacity against other individuals also acting in a private capacity. The government is not a party.

Which of the following part of the patient's medical record contains a numbered account of the patient's problems which helps to index documentation throughout the record? A. chief complaint B. initial plan C. problem list D. review of systems

C. problem list **The problem list serves as a table of contents for the patient record because it is filed at the beginning of the record and contains a numbered list of the patient's problems, which helps to index documentation throughout the record. The initial plan contains the strategy for managing patient care and any actions taken to investigate the patient's condition and to treat and educate the patient. The chief complaint, formally known as CC in the medical field, is a concise statement describing the symptom, problem, condition, diagnosis or other factor that is the reason for the encounter. The review of systems is a list of questions, arranged by organ system designed to uncover any dysfunction and/or disease.

In ICD-10-PCS, to code "freeing of vagus nerve root from surrounding scar tissue via open approach", the root operation would be: A. extirpation. B. detachment. C. release. D. division.

C. release. **Release is freeing a body part from an abnormal physical constraint. When the sole objective of the procedure is to free a body part without cutting into the body part, the root operation assignment should be release. Division is cutting into a body part without draining fluids and gases from the body in order to separate or transect a body part. Detachment is cutting off all or a portion of the upper or lower extremities. Extirpation is taking or cutting out solid matter from a body part.

The ability to obtain the same results from different studies using different methodologies and different populations is A. specificity. B. validity. C. reliability. D. confidence.

C. reliability. **In this question, one must know the definitions of validity and reliability. Validity is the degree to which scientific observations actually measure or record what they are supposed to measure. Reliability is the repeatability of scientific observations; can another person replicate the study using the same instrument and get the same results?

In ICD-10-PCS, to code "reduction of a displaced fraction and application of a cast", the root operation would be: A. insertion. B.change. C. reposition. D. immobilization.

C. reposition. **Reposition is moving to its normal location or other suitable location all or a portion of a body part. The reduction of a displaced fracture is coded to root operation reposition and the application of a cast in conjunction with the reposition procedure is not coded separately. Insertion is putting in a nonbiological appliance that monitors, assists, performs, or prevents a physiological function but does not physically take the place of a body part. Immobilization is not a root operation. Change is taking out or off a device from a body part and putting back an identical or similar device in or on the same body part without cutting or puncturing the skin or a mucous membrane.

A four-digit code that describes a classification of a product or service provided to a patient is a A. ICD-10-CM code. B. HCPCS Level II code. C. revenue code. D. CPT code.

C. revenue code. **All CPT codes have 5-digit numbers, HCPCS Level II codes have five alphanumeric characters, and ICD-10-CM codes do not report products or services.

Great Plains Transcription Services has a job opening for a home-based transcriptionist who can transcribe medical and surgical reports. Which of the following types of tests should be administered? A. intelligence B. stress C. skill D. personality

C. skill **Testing is a valuable tool often utilized during the employee selection process. A skills test should be administered to assess the applicant's skill in transcribing medical and surgical reports.

The healthcare facility has implemented an application that allows patients to support each other. The information system is A. continuity care document. B. portal. C. social media. D. personal health record.

C. social media. **Social media is being used by health care organizations for support groups and more.

According to the American Medical Association, medical decision making is measured by all of the following except the A. risk of complications. B. amount and complexity of data reviewed. C. specialty of the treating physician. D. number of diagnoses or management options.

C. specialty of the treating physician. **Medical decision making involves the complexity of establishing a diagnosis and/or selecting a management opinion or treatment plan as measured by the following: -Number of possible diagnoses and/or management options or treatment plans to be considered -Amount and/or complexity of the data to be obtained, reviewed, and analyzed -Risk of significant complications, morbidity, and/or mortality associated with the patient's presenting condition, diagnostic procedure(s), and/or the possible management options or treatment plans

The patient's heart is being monitored while he is at home. This is known as A. radio frequency identification. B. mobile device. C. telehealth. D. electronic health record.

C. telehealth. **The purpose of telehealth is to provide health care services to a patient when the health care provider is not face-to-face with the patient. Telehealth can take many forms including monitoring patients at home.

Research has shown that productivity increases with all of the following actions EXCEPT when A. it becomes the primary goal of management. B. it is measured. C. the office environment is reengineered. D. employees are rewarded for extra output.

C. the office environment is reengineered. **When productivity is measured, rewarded, and important to management, it increases. Research does not show that reengineering the office environment affects productivity.

A written consent from the patient is required from which of the following entities in order to learn a patient's HIV status? A. health care workers responsible for treating the patient B. Centers for Disease Control C. the patient's employer D. attending physician

C. the patient's employer **Specific written authorization that includes the purpose or need for information and a very detailed description of the extent or nature of the information to be disclosed regarding patient information for HIV/AIDS patients is required.

The name given to the error committed when the null hypothesis is rejected and it is actually true is A. alternative hypothesis. B. selection bias. C. type I error. D. type II error.

C. type I error. **The definition of the type I error is when the null hypothesis is rejected and it is actually true.

A mild, atypical lung infection with inflammation and fluid buildup, usually caused by Mycoplasma pneumoniae or other bacteria, in which the patient is mildly ill but does not require hospitalization and is treated with antibiotics is termed A. pneumoconiosis. B. emphysema. C. walking pneumonia. D. intrinsic asthma.

C. walking pneumonia. **Walking pneumonia is the least serious of the types of pneumonia. While still needing to be treated, patients can be somewhat active and do not need to be admitted to a hospital. Mild flu-like symptoms, cough, and slight chest pain are typical symptoms.

It has been decided that the coders will have access to all e-PHI in the EHR but they will not be able to add or edit data. This process is known as A. incidental uses. B. limited data set. C. workforce clearance procedure. D. information system activity review.

C. workforce clearance procedure. **Workforce clearance procedure is the process of determining what a user has access to in an information system and what they can do.

Nancy has asked the health care facility for a copy of her grandmother's health record. Her grandmother died 20 years ago. Nancy is not the executor of the estate, and she does not want to ask her aunt who is for permission. Select the appropriate response to Nancy. A. "You cannot access your grandmother's health record, as she has the right of privacy for 50 years after her death or her 100th birthday." B. "You cannot access your grandmother's health record until she has been deceased for 25 years." C. "Since you are a descendant, please sign this release." D. "You cannot access your grandmother's health record as she has the right of privacy for 50 years after her death."

D. "You cannot access your grandmother's health record as she has the right of privacy for 50 years after her death." **Her aunt, the executor, has to approve the request as her grandmother's right to privacy lasts for 50 years after her death.

Mrs. Jones had an appendectomy on November 1. She was taken back to surgery on November 2 for evacuation of a hematoma of the wound site. Identify the modifier that may be reported for the November 2 visit. A. -79 B. -58 C. -76 D. -78

D. -78 **Modifier -78 reflects circumstances when it is necessary for a patient to return to the operating room (unplanned) during the postoperative period. The procedure performed for the subsequent surgery is related to the initial procedure.

A study found that liver cancer rates per 100,000 males among cigarette smokers and nonsmokers in a major U.S. city were 48.0 and 25.4, respectively. The relative risk of developing liver cancer for male smokers compared to nonsmokers is A. 15.6. B. 48.0. C. 22.6. D. 1.89.

D. 1.89. **To calculate relative risk formula = Risk exposed / Risk not exposed = 48 / 25.4 = 1.89

The limiting charge is a percentage limit on fees specified by legislation that the nonparticipating physician may bill Medicare beneficiaries above the non-PAR fee schedule amount. The limiting charge is A. 50%. B.20%. C. 10%. D. 15%.

D. 15%. **This applies to NON-PAR physicians only.

A bundled payment that covers all services from immediately prior to a surgical procedure through the appropriate post-procedure care period is known as __________. A. capitation B. a prospective payment system C. fee-for-service D. a global payment

D. a global payment **Global because it covers the entire event.

Selena works 40 hours per week at Rio Grande Radiology, which pays time-and-a-half for overtime and double-time for holidays. During the past week, Selena took 6 hours of unpaid personal leave and worked an 8-hour holiday. How many hours will Selena be paid? A. 50 B. 48 C. 34 D. 42

D. 42 **Selena took 6 hours unpaid leave (40 - 6 = 34) but worked a holiday at double-time (8 x 2 = 16). Because 8 hours of the holiday are already figured in the workweek, add an additional 8 hours for holiday pay. So, 34 + 8 = 42.

Terminally ill patients with life expectancies of ______ may opt to receive hospice services. A. 6 months to a year B. 1 year or more C. 1 year or less D. 6 months or less

D. 6 months or less **Medicare requires a life expectancy of 6 months or less to receive hospice care.

The Healthcare Cost and Utilization Project (HCUP) consists of a set of databases that include data on inpatients whose care is paid for by third-party payers. HCUP is an initiative of the A. Centers for Medicare and Medicaid Services. B. National Library of Medicine. C. World Health Organization. D. Agency for Healthcare Research and Quality.

D. Agency for Healthcare Research and Quality. **A major initiative for the Agency for Healthcare Research and Quality (AHRQ) has been the Healthcare Cost and Utilization Project (HCUP). HCUP uses data collected at the state level from either claims data from the UB-04 or discharge-abstracted data, including UHDDS items reported by individual hospitals and, in some cases, by freestanding ambulatory care centers.

Margaret Vargas needs to have her mitral valve replaced. Her surgeon will discuss which of the following issues with her before the surgery? A. A biological valve (usually porcine) will last 10-15 years. B. A mechanical valve increases the risk of blood clots that can cause stroke. C. A mechanical valve will require that she take a "blood thinner" for the rest of her life. D. All answers apply.

D. All answers apply. **Because the artificial valves can contribute to increased blood clotting, an anticoagulant is prescribed for the patient. This is not an issue with biological replacement parts.

When logging into a system, you are instructed to enter a string of characters. These characters appear distorted onscreen, however. What kind of access control is this? A. token B. two-factor authentication C. biometrics D. CAPTCHA

D. CAPTCHA **CAPTCHA is a security measure used to ensure that a human is initiating the action. A series of letters and numbers are displayed that are distorted but still readable. The user must enter these characters to proceed.

Which of the following systems facilitates capturing standardized data with the electronic documentation of patient at the point of care? A. Continuity of Care Document (CCD) B. Clinical Documentation Architecture (CDA) C. Continuity of Care Record (CCR) D. Clinical Care Classification (CCC)

D. Clinical Care Classification (CCC) ** The Clinical Care Classification (CCC) system is two interrelated taxonomies: the CCC of Nursing Diagnoses and Outcomes and the CCC of Nursing Interventions and Actions that provide a standardized framework for documenting patient care in hospitals, home health agencies, ambulatory care clinics, and other health care settings. The CCC system can be used for a number of purposes. Primarily, it facilitates capturing standardized data with the electronic documentation of patient care at the point of care.

______ classifies inpatient hospital cases into groups that are expected to consume similar hospital resources. A. CMS B. IPPS C. MAC D. DRG

D. DRG **Diagnosis-related groups (DRGs) cluster reimbursement amounts based on the diagnosis and standards of care for the provision of procedures, services, and treatments.

Regarding hospital emergency department and hospital outpatient evaluation and management CPT code assignment, which statement is true? A. Each facility must use acuity sheets with acuity levels and assign points for each service performed. B. The level of service codes reported by the facility must match those reported by the physician. C. Each facility must use the same methodology used by physician coders based on the history, examination, and medical decision-making components. D. Each facility is accountable for developing and implementing its own methodology.

D. Each facility is accountable for developing and implementing its own methodology. **The facility must coordinate acceptance of the guidelines and how to follow them.

A website, available to the general public, permits anyone to review specific measures of quality care for a particular hospital, is called ___________ A. WebMD B. National Quality Measures Clearinghouse C. Agency for Healthcare Research and Quality D. Hospital Compare

D. Hospital Compare **A website, available to the general public, permits anyone to review specific measures of quality care for a particular hospital, is called Hospital Compare.

Explain the designated record set. A. It contains only billing information. B. It includes only demographic information. C. It contains only medical information. D. It contains both medical and billing information.

D. It contains both medical and billing information. **The designated record set is defined by HIPAA. It has medical, billing enrollment, payment, and other information.

You have recently been hired as the medical staff coordinator at your local hospital. Which database/registry will you utilize most often? A. MEDPAR B. Trauma Registry C. LOINC D. National Practitioner Data Bank (NPDB)

D. National Practitioner Data Bank (NPDB) **The National Practitioner Data Bank (NPDB) is a database of medical malpractice payments; adverse licensure actions including revocations, suspensions, reprimands, censures, probations, and surrenders of licenses for quality-of-care purposes only; and certain professional review actions (such as denial of medical staff privileges) taken by health care entities such as hospitals against physicians, dentists, and other health care providers.The law requires health care facilities to query the NPDB as part of the credentialing process when a physician initially applies for medical staff privileges and every two years thereafter.

____ are errors in medical care that are clearly identifiable, preventable, and serious in their consequences for patients. A. Potential compensable events B. Adverse preventable events C. Misadventures D. Never events or sentinel events

D. Never events or sentinel events **These are known as "Never" events because they should never happen. An example is a surgical instrument left inside a patient.

You have been given a list of data elements that need to be added to the database that you are creating. You notice, that on of the data elements is the patient's full name. Determine how to modify the data element in order to improve search capabilities. A. Utilize a drop down box field. B. Change the settings. C. Identify the table's key. D. Normalize the data element

D. Normalize the data element **Normalization would be required to break the field into last name, first name and middle initial (or name) so that the user can better search.

CMS published a final rule indicating a compliance date to implement ICD-10-CM and ICD-10-PCS. The use of these two code sets was effective on A. January 1, 2015. B. October 1, 2014. C. January 1, 2014. D. October 1, 2015.

D. October 1, 2015. **The U.S. government modified the International Classification of Diseases, 10th Revision, Clinical Modification (ICD-10-CM) for the reporting of morbidity data and reimbursement in the United States. In 2008, the Department of Health and Human Services (HHS) published a notice of proposed rulemaking that identified the replacement of ICD-9-CM with ICD-10-CM for diagnosis coding and ICD-10-PCS (International Classification of Diseases, 10th Revision, Procedure Coding System) for inpatient procedure coding. In 2008, the Department of Health and Human Service (HHS) published a notice effective October 1, 2013. However, the implementation was delayed twice and the new effective date for ICD-10 was set as October 1, 2015.

Currently, the enforcement of HIPAA Privacy and Security Rules is the responsibility of the A. Office of Inspector General. B. Department of Recovery Audit Coordinators. C. FBI. D. Office for Civil Rights.

D. Office for Civil Rights. **The Office for Civil Rights (OCR) ensures equal access to certain health and human services and protects the privacy and security of health information.

The FOCUS PDCA model used in performance improvement is best known for its change strategy technique of A. Business Process Engineering. B. Input, Through-Put, Output. C. Cause and Effect Diagramming. D. Plan, Do, Study, Act.

D. Plan, Do, Study, Act. **PDCA = Plan, Do, Check, Act

Which of the following classification systems was designed with electronic systems in mind and is currently being used for problem lists, ICU monitoring, patient care assessments, data collection, medical research studies, clinical trials, disease surveillance, and images? A. GEM B. SNDO C. ICDPC-2 D. SNOMED CT

D. SNOMED CT **One of the greatest strengths of SNOMED is that it was designed with electronic systems in mind. In addition to EHR applications, SNOMED CT is being used for problem lists, intensive care unit monitoring, decision support applications, alerts and reminders, patient care assessments, data collection at the point of care, medical research studies, clinical trials, disease surveillance, and image indexing. It is used in quality and public health reporting of infectious diseases, cancer, and biosurveillance. SNOMED CT enables patients and providers to describe an individual's health and care with true fidelity to the clinical status while maintaining the integrity of the information of both human and machine readability.

The standard claim form used by hospitals to request reimbursement for inpatient and outpatient procedures performed or services provided is called the A. CMS-1491. B. CMS-1600. C. CMS-1500. D. UB-04.

D. UB-04. **The UB-04 is used by hospitals. The CMS-1500 is used by physicians and other noninstitutional providers and suppliers. The CMS-1491 is used by ambulance services.

A patient undergoes outpatient surgery. During the recovery period, the patient develops atrial fibrillation and is subsequently admitted to the hospital as an inpatient. The present on admission (POA) indicator is A. U = documentation is insufficient to determine if condition was present at the time of admission. B. W = provider is unable to clinically determine if condition was present at the time of admission. C. N = not present at the time of inpatient admission. D. Y = present at the time of inpatient admission.

D. Y = present at the time of inpatient admission. **The atrial fibrillation developed prior to a written order for inpatient admission; therefore, it was present at the time of inpatient admission.

A patient signed an authorization to release information to a physician but decided not to go see that physician. Can he stop the release? A. Yes, as long as the physician agrees. B. Yes—in all circumstances. C. No, once the release is signed, it cannot be reversed. D. Yes, as long as it has not been released already.

D. Yes, as long as it has not been released already. **Authorizations can be revoked as long as the protected health information has not already been released.

You have been asked if digital signatures utilize encryption. Your response should be A. No, encryption is not a part of digital signatures. B. No, but digitized signatures utilize encryption. C. No, but electronic signatures utilize encryption. D. Yes, digital signature utilizes encryption.

D. Yes, digital signature utilizes encryption. **There are several types of signatures that can be used with varying levels of security. The digital signature uses encryptions which makes it the most secure.

At the request of one physician, a second physician provides advice regarding the evaluation and management of a specific problem. This is called A. risk factor intervention. B. a referral. C. concurrent care. D. a consultation.

D. a consultation. **A consultation is a type of service provided by a physician at the request of another physician or appropriate source to either recommend care for a specific condition or problem or to determine whether to accept responsibility for ongoing management of the patient's entire care or for the care of a specific condition or problem.

Which of the following conditions is typically treated with the insertion of a pacemaker? A. third-degree heart block B. Sick-sinus syndrome C. bradyarrhythmias D. all answers apply

D. all answers apply **Pacemakers are used to treat arrhythmias, which are problems with the rate or rhythm of the heartbeat.

Etiologies of dementia include A. brain tumors. B. ischemia. C. trauma. D. all answers apply.

D. all answers apply. **Alzheimer's is the most common cause of dementia, but all of these other conditions also contribute to the list of causes.

Centric Medical Center established a business associate agreement with Quenlinks Solutions to provide mobile devices for their physicians and nurses to enter patient information into the organization's EHR in real-time. Which of the following should be considered as a best practice for the use of mobile technology in this given scenario? A. safeguards and techniques for adequate protection of ePHI B. identification of device ownership C. required authorization for mobile technology use D. all of these answers apply

D. all of these answers apply **Best practices for the use of mobile technology within a health care organization or business associate should include identification of device ownership, personal device use within the organization, required authorization for mobile technology use, conditions under which ePHI is allowed on mobile devices, acceptable behaviors and use of ePHI on mobile technology, safeguards and techniques for adequate protection of ePHI, procedures for reporting lost or stolen devices, and evaluation and scanning of mobile technology on a regular basis.

The project manager is responsible for all of the following functions EXCEPT A. recommending plan revisions. B. creation of project plan. C. recruitment of project team. D. approval of the budget for the project.

D. approval of the budget for the project. **The project sponsor approves the budget for the project.

Select the speech recognition system where the staff dictates and then the editing of the dictation is done by editors. A. front-end speech recognition B. key-word spotting C. hidden Markov model D. back-end speech recognition

D. back-end speech recognition **Back-end speech recognition is when the physician dictates the discharge summary or other document and an editor reviews the document to ensure it is correct.

The information system strategic plan should be based on the A. proposed law B. the preferences of the employees C. works with the current technology D. business strategic plan

D. business strategic plan **While all of these should be considered when selecting an information systemm it should be based on the business strategic plan so that it can help meet the business plan.

The Correct Coding Initiative (CCI) edits contain a listing of codes under two columns titled "comprehensive codes" and "component codes." According to the CCI edits, when a provider bills Medicare for a procedure that appears in both columns for the same beneficiary on the same date of service, A. code both the comprehensive code and the component code. B. do not code either one. C. code only the component code. D. code only the comprehensive code.

D. code only the comprehensive code. **This is because the comprehensive code already includes the component service. Therefore, reporting both is redundant and overcoding.

Your job is to teach patients to use the patient portal. Your specialization is A. standards development. B. privacy. C. HIM management. D. consumer health informatics

D. consumer health informatics **Consumer health informatics includes PHRs, health literacy, and patient portals.

You are interested in performing some data analysis on patients with cardiac problems. You have downloaded the data that you need on the cardiology patients from the data warehouse into a smaller database that you can work with. You are using a(n) A. specialized data warehouse. B. executive information system. C. clinical data repository. D. data mart.

D. data mart. **A data mart is a specialized data warehouse. It contains a subset of data specifically for a purpose such as patient satisfaction.

Select the component of the database design where you will find the structures in the database, the types of records stored, and the fields used. A. database design specification B. database management C. data dictionary D. data model

D. data model **The data model is the design of the database. It contains the logical and physical data models that show the data and how it is stored.

A coder notices that some patient records contain incomplete documentation. The coder brings these records to the attention of the coding supervisor who will implement the next stage of the coding compliance program. What stage of coding compliance is being described in this scenario? A. correction B. prevention C. verification D. detection

D. detection **Detection is the process of identifying potential coding compliance problems. Correction is based on the review of patient records that contain potential coding compliance problems, during which specific compliance issues are identified and problem-solving methods are used to implement necessary improvements/corrections. Prevention involves educating coders and providers so as to prevent coding compliance problems from recurring. Verification provides an audit trail that the detection, correction, and prevention functions of the coding compliance program are being actively performed.

In ensuring security compliance, the Department of Health and Human Services has a seven-step guide to implementing a security management process. All of the following are steps within this processes EXCEPT A. develop an action plan. B. manage and mitigate risks. C. lead your culture, select your team, and learn. D. develop a contingency plan.

D. develop a contingency plan. **Security compliance is an ongoing process that requires constant analysis. According to the Department of Health and Human Services, the seven-step guide to implementing a security management process should include lead your culture, select your team, and learn; document your processes, findings and actions; review existing security of ePHI (perform security risk analysis); develop an action plan; manage and mitigate risks; attest for meaningful use security-related objectives; and monitor, audit, and update security on an ongoing basis.

Which of the following tubes conveys sperm from the seminal vesicle to the urethra? A. oviduct B. vas deferens C. epididymis D. ejaculatory duct

D. ejaculatory duct **Each ejaculatory duct is 2 cm long and is created when the seminal vesicle's duct merges with the vas deferens. At this point, these ducts travel through the prostate and drain into the urethra.

The high cost of staff turnover is eating away at the department's budget, so Thomas is creating a(n) __________ program to increase staff satisfaction. A. monitoring B. marketing C. continuing education D. employee retention

D. employee retention **Employee retention programs are used to ensure that high-quality staff members will continue to work at the organization.

All of these are quality improvement strategies EXCEPT A. computer-based reminder alerts B. performance-based bonuses C. providing patient education D. error-based penalties

D. error-based penalties **Quality Improvement Strategies use positive reinforcement and support to increase productivity and accuracy.

Integration of decision support systems and best practices in medicine is known as the practice of A. case management. B. subjective determination. C. outcome measures. D. evidence-based medicine.

D. evidence-based medicine. **Evidence-based medicine is the foundation of clinical decision support systems.

ICD-10-PCS utilizes the third character in the Medical and Surgical section to identify the "root operation." The name of the root operation that describes "cutting out or off, without replacing a portion of a body part" is A. removal. B. destruction. C. extirpation. D. excision.

D. excision. **Excision is the cutting out or off, without replacement, a portion of a body part. Extirpation is taking or cutting out solid matter from a body part. Destruction is physical eradication of all or a portion of a body part by the direct use of energy, force, or a destructive agent. Removal is taking out or off a device from a body part by cutting or by use of force.

You need a report that includes only Medicare patients. What operation enables you to generate this report? A. calculations B. grouping C. sorting D. filtering

D. filtering **Filtering is the operation that allows only a subset of data to be included.

Which of the following is incorrect about control charts? A. enables the organization of large numbers of ideas and prioritize them B. identified cause and effect of concerns C. itemizes all steps within a process for analysis D. focuses attention on process variation over a time period

D. focuses attention on process variation over a time period **Control charts are used for the recognition of variations.

In which brain lobe is the processing of emotions, intellect, and personality performed? A. occipital B. temporal C. parietal D. frontal

D. frontal **The frontal area of the brain is where each of the paired lobes (frontal lobes) is lying immediately behind the forehead. These frontal lobes include areas concerned with the processing of behavior, learning, personality, and voluntary movement.

Dashboards are a method used in health care that A. serve as a documentation format in patient records. B. serve as a teaching tool for third-party auditors. C. illustrate the marketing plan. D. graphically display a performance improvement project conducted.

D. graphically display a performance improvement project conducted. **A dashboard can provide a graphic overview of an entire project.

State Medicaid programs are required to offer medical assistance for A. all individuals age 65 and over. B. patients with end-stage renal disease. C. patients with a permanent disability. D. individuals with qualified financial need.

D. individuals with qualified financial need. **Patients 65 and older, ESRD, and those with permanent disabilities are covered under Medicare, not Medicaid.

Which of the following is needed when a physician conducts an invasive procedure? A. living will B. advanced directive C. general consent D. informed consent

D. informed consent **When a treatment or procedure is considered risky or invasive, a signed informed consent is required. The informed consent process is completed to ensure the patient has a basic understanding of diagnosis, the nature of the treatment or procedure, along with the risks, benefits, and alternatives, including opting out of the treatment. The informed consent process also identifies the individuals who will be providing the treatment or performing the procedure, and offers an opportunity for the patient to ask questions.

The ideal consent for medical treatment obtained by the physician is A. implied. B. verbal. C. expressed. D. informed.

D. informed. **The informed consent is the ideal consent because within this process, the patient obtains a basic understanding of the diagnosis, the nature of the treatment, or procedure. With an informed consent, the patient has an understanding of the risks, benefits, and alternatives to the treatment that is being recommended. With an informed consent, the patient is made aware of who will be performing the treatment or procedure, and it provides an opportunity for patients to ask questions and receive answers.

The foundation for communicating all patient care goals in long-term care settings is the A. cognitive assessment. B. Uniform Hospital Discharge Data Set. C. legal assessment. D. interdisciplinary plan of care.

D. interdisciplinary plan of care. **Unlike the acute care hospital, where most health care practitioners document separately, the patient care plan is the foundation around which patient care is organized in long-term care facilities because it contains the unique perspective of each discipline involved.

Scabies, a highly contagious condition that produces intense pruritus and a rash, is caused by A. pediculosis capitis. B. candidiasis. C. ringworm. D. itch mites.

D. itch mites. **Scabies is caused by an infestation by the itch mite, Sarcoptes scabiei.

To enter the results of a urinalysis into the computer system, you would use a(n) A. radiology system. B. order entry/results reporting system. C. pharmacy system. D. laboratory system.

D. laboratory system. **The laboratory information system manages the lab and stores laboratory test result data such as a urinalysis.

John is a 45-year-old male who is mentally disabled. Who can authorize release of his health record? A. executive of his will B. John C. John's sister D. legal guardian

D. legal guardian **Even though John is of age, he is mentally incompetent and therefore requires a guardian to sign the release.

The family of Mr. Kent filed a wrongful death lawsuit against Pier Memorial Hospital 10 years after the procedure was performed. In order to avoid the risk of Mr. Kent's electronic medical record being destroyed, a _______ was ordered by the court. A. spoliation hold B. restitution waiver C. subpoena duces tecum D. legal hold

D. legal hold **A legal hold is a communication issued because of current or anticipated litigation, audit, government investigation, or other such matters that suspend the normal disposition or processing of records.

A 19-year-old college student, who lives on campus in a dormitory, is brought to the ER by his roommates, complaining of a severe headache, nuchal rigidity, fever, and photophobia. The ER physician performs an LP and orders a CSF analysis with a bacterial culture and sensitivity. The young man is admitted to the ICU with a provisional diagnosis of A. a group A beta-hemolytic streptococcal throat infection. B. tuberculosis with drug-resistant Mycobacterium tuberculosis-positive sputum. C. a methicillin-resistant Staphylococcus aureus skin infection. D. meningitis due to Neisseria meningitidis-positive cerebrospinal fluid.

D. meningitis due to Neisseria meningitidis-positive cerebrospinal fluid. **Meningitis is a contagious, life-threatening condition that can cause brain damage. It can be caused by a virus, fungus, or bacteria. Neisseria meningitidis is a typical bacterial causative agent.

All of the following are examples of intentional torts EXCEPT A. invasion of privacy. B. assault and battery. C. defamation of character. D. misfeasance.

D. misfeasance. **Intentional torts occur when a person commits an act purposefully knowing that harm would likely occur. Intentional torts include assault, battery, false imprisonment, defamation of character, fraud, invasion of privacy, and infliction of emotional distress.

Which valve is between the left atrium and left ventricle? A. aortic B. tricuspid C. pulmonary D. mitral

D. mitral **The mitral valve, also known as the bicuspid valve or left atrioventricular valve, is a valve with two flaps in the heart, which lies between the left atrium and the left ventricle.

The purpose of the notice of privacy practices is to A. notify researchers of allowable data use. B. report incidents to the OIG. C. notify the patient of audits. D. notify the patient of uses of PHI.

D. notify the patient of uses of PHI. **The purpose of the notice of privacy practices is to notify the patient how the covered entity will use the PHI and what the patient's rights are related to PHI.

The database being implemented must be able to store video, images, and audio. Which of the following database models should be considered? A. hierarchical B. network C. relational D. object oriented

D. object oriented **The object-oriented database is designed to store objects that include video, audio, and images.

Employee turnover is expensive and stressful on staff and reflects poorly on managers. The best defense against employee dissatisfaction is A. the employee handbook. B. weekly departmental meetings. C. written policies and procedures. D. open and honest communication.

D. open and honest communication. **Weekly department meetings, having an employee handbook, and written policies and procedures are all part of the necessary open and honest communication.

The director of a Health Information Department prepared a tool in which the following information could be obtained: job title, reporting line, span of control, and routes of promotion. The document she prepared is a(n) A. job procedure. B. work distribution chart. C. job description. D. organizational chart.

D. organizational chart. **The organizational chart graphically represents the formal structure of an organization.

A coding supervisor trained the employees that they should code signs and symptoms in addition to the established diagnosis code. This is an example of: A. upcoding. B. jamming. C. unbundling. D. overcoding.

D. overcoding. **Overcoding is reporting codes for signs and symptoms in addition to the established diagnosis code. Unbundling is reporting multiple codes to increase reimbursement when a single combination code should be reported. Upcoding is reporting codes that are not supported by documentation in the patient record for the purpose of increasing reimbursement. Jamming is routinely assigning an unspecified ICD-9-CM or ICD-10-CM disease code instead of reviewing the coding manual to select the appropriate code.

This document is published by the Office of Inspector General (OIG) every year. It details the OIG's focus for Medicare fraud and abuse for that year. It gives health care providers an indication of general and specific areas that are targeted for review. It can be found on the Internet on CMS's website. A. the OIG's Evaluation and Management Documentation Guidelines B. the OIG's Model Compliance Plan C. the Federal Register D. the OIG's Work Plan

D. the OIG's Work Plan **The OIG Work Plan, published every year, itemizes specific services that are, or will be, under investigation.

While data governance focuses primarily on managing data as it is being created within a healthcare system, information governance focuses instead on managing A. data currency B. the granularity of healthcare systems C. data accuracy D. the output of those systems

D. the output of those systems **Data governance manages the information created in the different systems used in healthcare, while information governance manges the information output from those systems.

In CPT, Category III codes include codes A. or supplies, drugs, and durable medical equipment. B. to measure performance. C. for use by nonphysician practitioners. D. to describe emerging technologies.

D. to describe emerging technologies. **CPT Category III includes temporary codes that represent emerging medical technologies, services, and procedures that have not yet been approved for general use by the FDA and are not otherwise covered by CPT codes. It gives physicians and researchers a system for documenting the use of unconventional methods so that their efficacy and outcomes can be tracked.

When a provider bills separately for procedures that are a part of the major procedure, this is called A. discounting. B. fraud. C. packaging. D. unbundling.

D. unbundling. **Unbundling is, instead of reporting the one combination or comprehensive code, it is taken apart and reported with individual codes for the components.

What follow-up rate does the American College of Surgeons mandate for all cancer cases to meet approval requirements as a cancer program? A. 90% B. 70% C. 100% D. 80%

A. 90% **A 90% target rate. This target rate has been established by the Commission on Cancer (CoC) of the American College of Surgeons (ACoS).

The director of the HIM Department is explaining incentives to physicians for entering their clinical documentation in the electronic health record. Which of the following would be the key advantage in using this type of data entry? A. Enhanced databases will provide information for improved clinical care. B. Those physicians not in compliance will be denied admitting privileges. C. Multiple users will not have access to the same information simultaneously. D. Training will be offered by the hospital.

A. Enhanced databases will provide information for improved clinical care. **There are many advantages. When speaking with physicians, the HIM director should stress how enhanced databases can improve the provision of patient care.

The Assistant Director of Record Processing is evaluating software packages for a chart-tracking system in the HIM Department. What is the BEST method to verify that the software will work as marketed? A. Test the software prior to purchase. B. Perform a vendor reference check. C. Read consumer reports before buying. D. Visit corporate headquarters of the vendor.

A. Test the software prior to purchase. **Testing is the only way to confirm that the software will work as advertised.

As supervisor of the cancer registry, you report the registry's annual caseload to administration. The most efficient way to retrieve this information would be to use A. accession register. B. patient index. C. patient abstracts. D. follow-up files.

A. accession register. **The accession register is a permanent log of all the cases entered into the database. Each number assigned is preceded by the accession year, making it easy to assess annual workloads.

What is the chief criterion for determining record inactivity? A. amount of space available for storage of newer records B. preference of the medical staff C. efficiency of microfilming D. Medicare's definition of inactivity

A. amount of space available for storage of newer records **Records may need to be purged to create space for newer records. When purging records, the HIM professional must consider many factors, such as the facility's readmission rate, available file space, and user access needs.

University Hospital, a 900-bed tertiary health care organization, is undergoing an information systems development. What system would best meet its needs? A. application service provider model B. legacy system C. clinical workstation D. IBM Medical Information Systems Program

A. application service provider model **With the application service provider model, another entity or business manages the technology, not the health care organization; and the software and servers are located off-site.

Which one of the following is NOT a technical security control employed by electronic health record systems? A. automatic log off B. data encryption protocols C. audit trails D. user-based access controls

A. automatic log off **Automatic log off after a period of inactivity is an administrative safeguard. Technical safeguards consist of five categories: access controls, audit controls, integrity, person or identity authentication, and transmission security.

Many of the principles of forms design apply to both paper-based and computer-based systems. For example, the physical layout of the form and/or screen should be organized to match the way the information is requested. Facilities that are scanning and imaging paper records as part of a computer-based system must give careful consideration to A. bar code placement. B. placement of hospital logo. C. use of box design. D. signature line for authentication.

A. bar code placement. **Most facilities use bar-coded patient identification to ensure proper indexing into the imaging system.

A 16-year-old female delivers a stillborn infant in Mercy Hospital. The clinical documentation on the stillborn infant would A. be filed in the mother's record. B. not be retained in hospital records. C. be retained in a separate file in the administrative office. D. be filed in a health record created for the infant.

A. be filed in the mother's record. **Stillborns are also called fetal deaths. The documentation on the stillborn is filed in the mother's health record. The stillborn has no health record.

Documentation found in acute care health records should include core measure quality indicators required for compliance with Medicare's Health Care Quality Improvement Program (HCQIP). A typical quality indicator for patients with pneumonia might be A. blood culture before first antibiotic received. B. discharged on antithrombotic. C. early administration of aspirin. D. beta blocker at discharge.

A. blood culture before first antibiotic received. **"Beta blocker at discharge" and "early administration of aspirin" represent typical quality indicators for patients with acute myocardial infarction; "discharged on antithrombotic" represents a quality indicator typical for stroke patients. The correct answer is "blood culture before first antibiotic received" since this is the only clinical indicator that applies directly to pneumonia patients.

It is recommended that all but which of the following information should be permanently retained in some format, even when the remainder of the health record is destroyed? A. dates of admission, discharge, and encounters B. discharge summaries C. nursing notes D. physician names

A. dates of admission, discharge, and encounters **Although most of the medical record information can be destroyed after a certain time constraint, this information is permanently retained in the facility's master patient index (MPI).

A major contribution to a successful CDI program is the ability of the CDI specialist to demonstrate to the medical staff as well as to administration the powerful impact that precise documentation has on the internal and external data reporting. In this role, he/she is acting as a(n) A. educator. B. reviewer. C. ambassador. D. manager.

A. educator. **The CDI professional may act as a reviewer and manager, but the duties described are most representative of his/her role as an educator.

As the chair of a Forms Review Committee, you need to track the field name of a particular data field and the security levels applicable to that field. Your best source for this information would be the A. facility's data dictionary. B. MDS. C. UHDDS. D. glossary of health care terms.

A. facility's data dictionary. **"MDS" and "UHDDS" are types of data sets for collecting data in long-term (MDS) and acute care (UHDDS) facilities. A data dictionary should include security levels for each field as well as definitions for all entities.

Electronic health record built-in tools that can make data capture easier include all but which one of the following? A. flow process charts B. automated quality measures C. data dictionaries D. clinical decision support systems

A. flow process charts **A flow process chart depicts a sequence of actions for a particular process. It is essential in the continuous improvement process.

In the computerization of forms, good screen-view design, along with the options of alerts and alarms, makes it easier to ensure that all essential data items have been captured. One essential item to be captured on the physical exam is the A. general appearance as assessed by the physician. B. subjective review of systems. C. chief complaint. D. family history as related by the patient.

A. general appearance as assessed by the physician. **The medical history (including chief complaint, history of present illness, past medical history, personal history, family history, and a review of systems) is provided by the patient or the most knowledgeable available source. The physical examination adds objective data to the subjective data provided by the patient. This exam begins with the physician's objective assessment of the patient's general condition.

As a new HIM manager of an acute care facility, you have been asked to update the facility's policy for a physician's verbal orders in accordance with Joint Commission standards and state law. Your first area of concern is the qualifications of those individuals in your facility who have been authorized to record verbal orders. For this information, you will consult the A. hospital bylaws, rules, and regulations. B. Federal Register. C. consolidated manual for hospitals. D. policy and procedure manual.

A. hospital bylaws, rules, and regulations. **Although Joint Commission, CMS, and state laws may include standards for verbal orders, the specific information regarding which employees have been given authority to transcribe verbal orders in your facility should be located in your hospital's bylaws, rules, and regulations.

A qualitative review of a health record reveals that the history and physical for a patient admitted on June 26 was performed on June 30 and transcribed on July 1. Which of the following statements regarding the history and physical is true in this situation? Completion and charting of the H&P indicates A. noncompliance with Joint Commission standards. B. compliance with Joint Commission standards. C. compliance with Medicare regulations. D. compliance with Joint Commission standards for nonsurgical patients.

A. noncompliance with Joint Commission standards. **Joint Commission specifies that H&Ps must be completed within 24 hours or prior to surgery.

The steps in developing a record retention program include all but which of the following? A. notifying the courts of the destruction B. determining the storage format and location C. destroying records that are no longer needed D. assigning each record a retention period

A. notifying the courts of the destruction **Notifying the courts is not required for record destruction.

For a health care facility to meet its document destruction needs, the certificate of destruction should include all but which of the following elements? A. patient notification B. location of destruction C. acceptance of fiduciary responsibility D. unique and serialized transaction number

A. patient notification **Patient notification is not required for document destruction.

Reviewing a medical record to ensure that all diagnoses are justified by documentation throughout the chart is an example of A. qualitative review. B. quantitative review. C. peer review. D. legal analysis.

A. qualitative review. **Peer review typically involves quality of care issues rather than quality of documentation issues. Legal analysis ensures that the record entries would be acceptable in a court of law. The correct choice is qualitative review.

Though you work in an integrated delivery network, not all systems in your network communicate with one another. As you meet with your partner organizations, you begin to sell them on the concept of an important development intended to support the exchange of health information across the continuum within a geographical community. You are promoting that your organization join a A. regional health information organization. B. data retrieval portal group. C. data warehouse. D. continuum of care.

A. regional health information organization. **Regional health information organizations are intended to support health information exchange within a geographic region.

Which of the following is NOT a factor to consider when developing a record retention program? A. reimbursement guidelines B. cost of space to maintain paper records C. record usage in the facility determined by health care provider activity D. legal requirements as determined by statute of limitations

A. reimbursement guidelines **It is not necessary to consider reimbursement guidelines when developing a record retention program.

The minimum length of time for retaining original medical records is primarily governed by A. state law. B. medical staff. C. Joint Commission. D. readmission rates.

A. state law. **The statute of limitations for each state is information that is crucial in determining record retention schedules.

In your acute care facility it has become critical that up-to-date information regarding patients who are transferred to the oncology patient care unit be immediately sent to an outpatient scheduling system to facilitate outpatient appointments. This transfer of service can be obtained most readily from A. the R-ADT system. B. generic screens used by record abstractors. C. the disease index. D. the indicator monitoring program.

A. the R-ADT system. **For tracking in-house patients who have been transferred to a specialty unit, the best source of information is the registration-admission, discharge, and transfer system.

In the past, Joint Commission standards have focused on promoting the use of a facility-approved abbreviation list to be used by hospital care providers. With the advent of the commission's national patient safety goals, the focus has shifted to the A. use of prohibited or "dangerous" abbreviations. B. flagrant use of specialty-specific abbreviations. C. prohibited use of any abbreviations. D. use of abbreviations in the final diagnosis.

A. use of prohibited or "dangerous" abbreviations. **As part of its National Patient Safety Goals initiative, the Joint Commission required hospitals to prohibit abbreviations that have caused confusion or problems in their handwritten form, such as "U" for unit, which can be mistaken for "O". Spelling out the word "unit" is preferred.

The final HITECH Omnibus Rule expanded some of HIPAA's original requirements, including changes in immunization disclosures. As a result, where states require immunization records of a minor prior to admitting a student to a school, a covered entity is permitted to dsclose proof of immunication to a school without A. written authorization of the parent B. any communication with the parent C. documentation of any kind D. written authorization by the child

A. written authorization of the parent **The "Disclosure of Student Immunizations to Schools" provision of the final rule permits a covered entity to disclose proof of immunization to a school (where state law requires it prior to admitting a student) without written authorization of the parent. An agreement must still be obtained and documented, but no signature by the parent is required.

How many years does the Food and Drug Administration require research records pertaining to cancer patients be maintained? A. 7 B. 30 C. permanently D. 5

B. 30 **Organizations with special patient populations (such as research patients) have to maintain their records for a longer period of time. The FDA requires research records pertaining to cancer patients to be maintained for 30 years.

Select the appropriate situation for which a final progress note may legitimately be substituted for a discharge summary in an inpatient medical record. A. Patient admitted with COPD 1/4/2018 and discharged 1/7/2018. B. Baby Boy Hiltz, born 1/5/2018, maintained normal status, discharged 1/7/2018. C. Baby Boy Doe admitted 1/3/2018, died 1/4/2018. D. Baby Boy Hiltz's mother admitted 1/5/2018, C-section delivery, and discharged 1/7/2018.

B. Baby Boy Hiltz, born 1/5/2018, maintained normal status, discharged 1/7/2018. **A final progress note may substitute for a discharge summary in the following cases: patients who are hospitalized less than 48 hours with problems of a minor nature, normal newborns, and uncomplicated obstetrical deliveries. "Patient admitted with COPD 1/4/2018 and discharged 1/7/2018" does not qualify because of the nature of the problem and the length of stay. "Baby Boy Hiltz's mother admitted 1/5/2018, C-section delivery, and discharged 1/7/2018" describes a complicated delivery, and "Baby Boy Doe admitted 1/3/2018, died 1/4/2018" cites a severely ill patient rather than one with a minor problem.

The best resource for checking out specific voluntary accreditation standards and guidelines for a rehabilitation facility is the A. Medical Staff Bylaws, Rules, and Regulations. B. CARF manual. C. Joint Commission manual. D. Conditions of Participation for Rehabilitation Facilities.

B. CARF manual. **The manual published by the Commission on Accreditation of Rehabilitation Facilities will have the most specific and comprehensive standards for a rehabilitation facility.

The HIM practitioner's duty to retain health information via the archiving and storage of health data includes all EXCEPT which of the following? A. a retention plan for multiple volumes of records B. a retention plan for financial data C. strategies ensuring that inactive records are as secure as active records D. strategies that consider accessibility, natural disasters, and innovations in storage technology

B. a retention plan for financial data **The HIM professional would not be responsible for retaining financial data.

The clinical laboratory department staff can use a database that allows them to see what laboratory tests were conducted and the results of those tests. By contrast, the billing department staff can only see that portion of the database that lists the laboratory tests that generate a charge, but they cannot see the test results. What kind of control is this an example of? A. concurrency B. access C. cost D. integrity

B. access **This access safeguard allows employees to only access the data that they need to perform their jobs.

Using a template to collect data for key reports may help to prompt caregivers to document all required data elements in the patient record. This practice contributes to data A. accuracy. B. comprehensiveness C. timeliness. D. security.

B. comprehensiveness **Data comprehensiveness refers specifically to the presence of all required data elements.

Which of the following is NOT considered a challenge in the adoption of an electronic health record system? A. design of the work flow and processes B. contribution to the quality of patient care C. individual state legal and regulatory issues D. physician willingness to adopt

B. contribution to the quality of patient care **The adoption of EHRs has faced many challenges over the years, such as work flow and process design, legal and regulatory issues, and physician adoption.

General Hospital utilizes various related files that include clinical and financial data to generate reports such as MS-DRG case mix reports. What application would be MOST effective for this activity? A. word processing B. database management system C. desktop publishing D. command interpreter

B. database management system **A database management system provides a better solution for capturing and managing data.

Which of the following is NOT a benefit of the electronic document management system in the HIM Department? A. online availability of information B. decreased use of computer technology C. system security and confidentiality D. multiuser simultaneous access

B. decreased use of computer technology **Utilization of an electronic document management system (EDMS) involves increased use of computer technology. An EDMS makes it possible to scan documents that originated on paper and perform indexing, storage, and retrieval functions electronically.

Which of the following is least likely to be identified by a retrospective quantitative analysis of a health record? A. X-ray report charted on the wrong record B. discrepancy between postoperative diagnosis by the surgeon and pathology diagnosis by the pathologist C. need for physician authentication of two verbal orders D. missing discharge summary

B. discrepancy between postoperative diagnosis by the surgeon and pathology diagnosis by the pathologist **"Missing discharge summary," "need for physician authentication of two verbal orders," and "X-ray report charted on the wrong record" all represent common checks performed by a quantitative analysis clerk: missing reports, signatures, or patient identification. "Discrepancy between postoperative diagnosis by the surgeon and pathology diagnosis by the pathologist" represents a more in-depth review dealing with the quality of the data documented.

Joint Commission does not approve auto authentication of entries in a health record. The primary objection to this practice is that A. it is too easy to delegate use of computer passwords. B. evidence cannot be provided that the physician actually reviewed and approved each report. C. electronic signatures are not acceptable in every state. D. tampering too often occurs with this method of authentication.

B. evidence cannot be provided that the physician actually reviewed and approved each report. **Auto authentication is a policy adopted by some facilities that allow physicians to state in advance that transcribed reports should automatically be considered approved and signed (or authenticated) when the physician fails to make corrections within a preestablished time frame (e.g., "Consider it signed if I do not make changes within 7 days."). Another version of this practice is when physicians authorize the HIM department to send weekly lists of unsigned documents. The physician then signs the list in lieu of signing each individual report. Neither practice ensures that the physician has reviewed and approved each report individually.

An example of a primary data source is the A. physician index. B. health record. C. hospital statistical report. D. cancer registry.

B. health record. **The health record is a primary source because it contains patient information documented by caregivers who participated in the care of the patient. Indices and registries are considered secondary data sources.

Which of the following is NOT a major management challenge in the storage and retention of electronic health record systems? A. ensuring that health information can be retrieved in a timely manner B. maintaining the paper-based storage system C. keeping technology updated in order to retrieve data D. following state and federal laws and accreditation requirements when developing retention and destruction policies

B. maintaining the paper-based storage system **Maintaining a paper-based storage system is not a challenge with electronic health record systems.

As part of Joint Commission's National Patient Safety Goal initiative, acute care hospitals are now required to use a preoperative verification process to confirm the patient's true identity and to confirm that necessary documents such as X-rays or medical records are available. They must also develop and use a process for A. apprising the patient of all complications that might occur. B. marking the surgical site. C. including the primary caregiver in surgery consults. D. including the surgeon in the preanesthesia assessment.

B. marking the surgical site. **The Joint Commission requires hospitals to mark the correct surgical site and to involve the patient in the marking process to help eliminate wrong site surgeries.

Setting up a drop-down menu to make sure that the registration clerk collects "gender" as "male, female, or unknown" is an example of ensuring data A. reliability. B. precision C. timeliness. D. validity.

B. precision **Validity refers to the accuracy of data, while reliability refers to its consistency. Timeliness refers to data being available within a time frame helpful to the user, and data precision refers to data that is precise and collected in its exact form so there will be no variability in the data.

During a retrospective review of Rose Hunter's inpatient health record, the health information clerk notes that on day 4 of hospitalization, there was one missed dose of insulin. What type of review is this clerk performing? A. utilization review B. qualitative review C. legal review D. quantitative review

B. qualitative review **Quantitative analysis involves checking for the presence or absence of necessary reports and/or signatures, while qualitative analysis may involve checking documentation consistency, such as comparing a patient's pharmacy drug profile with the medication administration record.

As a prerequisite in the implementation of an electronic health record, what process would facilitate automatic indexing? A. removing portions of the patient record that will not be scanned B. redesigning forms to include bar codes C. scanning only emergency room records initially D. converting all microfilm to optical disk format

B. redesigning forms to include bar codes **Barcoding is an essential component of the document indexing function.

Which of the following is NOT an advantage of an electronic document management system? A. workflow can be controlled electronically B. scanned record data can be manipulated C. user has immediate access to health record D. paper record can be stored digitally

B. scanned record data can be manipulated **The user cannot manipulate the data because the document is stored as an image.

As HIM director, you must ensure a means to regulate access and ensure preservation of data in the health care facility's computer system. Which of the following is NOT a security measure that can be implemented to prevent privacy violations in this computer system? A. authentication B. stonewall C. encryption D. disaster recovery plan

B. stonewall **A stonewall is not a security measure.

When health care facilities close or medical practices dissolve, procedures for disposition of patient records should take into consideration all of the following EXCEPT A. Medicare requirements. B. state laws and licensing standards. C. Communities of Practice requirements. D. needs and wishes of patients.

C. Communities of Practice requirements. **It is not necessary to consider Communities of Practice requirements when addressing disposition of patient records.

One of the Joint Commission National Patient Safety Goals (NSPGs) requires that health care organizations eliminate wrong-site, wrong-patient, and wrong-procedure surgery. In order to accomplish this, which of the following would NOT be considered part of a preoperative verification process? A. Review the medical records and/or imaging studies. B. Mark the surgical site. C. Follow the daily surgical patient listing for the surgery suite if the patient has been sedated. D. Confirm the patient's true identity.

C. Follow the daily surgical patient listing for the surgery suite if the patient has been sedated. **"Confirm the patient's true identity," "mark the surgical site," and "review the medical records and/or imaging studies"—these are usually in the protocol to prevent wrong site, wrong patient, or wrong surgery. The correct answer is following the daily surgical patient listing—that choice would NOT be an appropriate step in making sure you have the correct identity of the patient, the correct site, or the correct surgery.

What data cannot be retrieved from the MEDPAR? A. Charges broken down by specific types of services B. ICD-10-CM diagnosis codes C. Non-Medicare patient data D. Data on the provider

C. Non-Medicare patient data **The MEDPAR consists of Medicare claims data only.

Gerda Smith has presented to the ER in a coma with injuries sustained in a motor vehicle accident. According to her sister, Gerda has had a recent medical history taken at the public health department. The physician on call is grateful that she can access this patient information using the area's A. expert system. B. CPOE. C. RHIO. D. EDMS system.

C. RHIO. **With the increasing number of health care entities implementing EHR systems, the networking of electronic information between facilities has become a reality in some areas due to the establishment of regional health information organizations. EDMS = electronic data management system. CPOE = computerized provider order entry system

Which one of the following actions would NOT be included in the professional obligations of the health information practitioner that lead to responsible handling of patient health information? A. Extend privacy and security principles into all aspects of the data use, access, and control program adopted in the organization. B. Honor the patient-centric direction of the national agenda. C. Take a compromising position toward optimal interpretation of nonspecific regulations and laws. D. Educate consumers about their rights and responsibilities regarding the use of their personal health information.

C. Take a compromising position toward optimal interpretation of nonspecific regulations and laws. **Taking a compromising position regarding interpretation of regulations and laws would be unprofessional and unethical. Per the AHIMA Code of Ethics, AHIMA members should demonstrate ethics and compliance in their professional practice activities.

Unless state or federal laws require longer time periods, AHIMA recommends that patient health information for minors be retained for at least how long? A. 10 years after the age of majority B. 10 years after the most recent encounter C. age of majority plus statute of limitation D. permanently

C. age of majority plus statute of limitation **AHIMA has recommended retention standards for various types of health information. For minors, it is recommended that the retention period extend beyond the age of majority (some states is age 18, others 21 years of age). The record retention period recommended for these records is the age of majority plus the statute of limitation mandated for retention of adult records.

A health care facility has received a request to participate in a statewide study on cleft lip and cleft palate. This study would include data from the past year and subsequent years. Given that each of the data sources cited below contains the necessary information, the initial data would be most easily collected from the A. state bureau of vital statistics. B. newborn records. C. birth defects registry. D. maternal records.

C. birth defects registry. **Cleft lips and cleft palates are birth defects. This information would be included in a birth defects registry.

The assistant director of HIM is evaluating software that would use electronic logging of the location of incomplete and delinquent records as they move through the completion process. What departmental function is this most useful for? A. transcription B. coding C. chart tracking D. release of information

C. chart tracking **With an automated chart-tracking system, the record is "checked out" and assigned to the requestor. The record's location is maintained by the computer.

Which of the following is a secondary data source that would be used to quickly gather the health records of all juvenile patients treated for diabetes within the past 6 months? A. pediatric census sheet B. patient register C. disease index D. procedure index

C. disease index **The disease index is a listing of diagnosis code numbers that matches the codes to the patients who had those diseases.

You have been asked to identify every reportable case of cancer from the previous year. A key resource will be the facility's A. patient index. B. number control index. C. disease index. D. physicians' index.

C. disease index. **The major sources of case findings for cancer registry programs are the pathology department, the disease index, and the logs of patients treated in radiology and other outpatient departments. The number index identifies new health record numbers and the patients to whom they were assigned. The physicians' index identifies all patients treated by each doctor. The patient index links each patient treated in a facility with the health number under which the clinical information can be located.

Which of the following features should NOT be considered when designing screens to capture quality health data? A. a prompt for more information B. built-in alerts to notify users of possible errors C. left to right and bottom to top formatting D. the use of abbreviations on data fields

C. left to right and bottom to top formatting **It is not necessary to consider formatting because most EHRs come equipped with forms and templates.

When engaging the services of a paper record storage vendor, all but which one of the following factors should be included in the contract? A. confidentiality policies B. record retrieval turnaround time C. name and cell phone number of vendor delivery drivers D. climate control policies

C. name and cell phone number of vendor delivery drivers **The contract should address all aspects of storing, retrieving, and protecting the paper records. Identification and personal contact information of vendor employees should not be part of the contract.

In creating a new form or computer view, the designer should be most driven by A. QIO standards. B. flow of data on the page or screen. C. needs of the users. D. medical staff bylaws.

C. needs of the users. **The needs of the user are the primary concern in forms design.

The best example of point-of-care service and documentation is A. using an automated tracking system to locate a record. B. doctors using voice recognition systems to dictate radiology reports. C. nurses using bedside terminals to record vital signs. D. using occurrence screens to identify adverse events.

C. nurses using bedside terminals to record vital signs. **Of the processes listed, only "nurses using bedside terminals to record vital signs" pertains to the clinical application of data entry into the patient's record at the time and location of service.

One record documentation requirement shared by both acute care and emergency departments is A. time and means of arrival. B. problem list. C. patient's condition on discharge. D. advance directive.

C. patient's condition on discharge. **Time and means of arrival is required on ED records only. Evidence of known advance directive is required on inpatient records only. Problem list is typically required on ambulatory records only. The correct answer is "patient's condition on discharge," which should be recorded on both ED and acute care records.

Which of the following is a form or view that is typically seen in the health record of a long-term care patient but is rarely seen in records of acute care patients? A. medical consultation B. physical exam C. pharmacy consultation D. emergency record

C. pharmacy consultation **Pharmacy consults are required for elderly patients who typically take multiple medications. These consults review for potential drug interactions and/or discrepancies in medications given and those ordered.

Based on the following documentation in an acute care record, where would you expect this excerpt to appear? **The patient is alert and in no acute distress. Initial vital signs: T 98, P 102 and regular, R 20 and BP 120/69...** A. past medical history B. chief complaint C. physical exam D. social history

C. physical exam **"Past medical history," "social history," and "chief complaint" represent components of the medical history as supplied by the patient, while the physical exam is an entry obtained through objective observation and measurement made by the provider.

Which of the following is NOT a document input device in the electronic document management system? A. bar codes B. touch screen C. printer​ D. scanner

C. printer​ **A printer is an output device.

Which of the four distinct components of the problem-oriented record serves to help index documentation throughout the record? A. initial plan B. progress notes C. problem list D. database

C. problem list **In a POMR, the database contains the history and physical; the problem list includes titles, numbers, and dates of problems and serves as a table of contents of the record; the initial plan describes diagnostic, therapeutic, and patient education plans; and the progress notes document the progress of the patient throughout the episode of care, summarized in a discharge summary or transfer note at the end of the stay.

As the coding supervisor, your job description includes working with agents who have been charged with detecting and correcting overpayments made to your hospital in the Medicare Fee for Service program. You will need to develop a professional relationship with A. QIO physicians. B. MEDPAR representatives. C. recovery audit contractors. D. the OIG.

C. recovery audit contractors. **The correct answer is "recovery audit contractors." The RAC program is mandated to find and correct improper Medicare payments paid to health care providers participating in the Medicare reimbursement program. OIG stands for Office of Inspector General; MEDPAR stands for Medicare Provider Analysis and Review; QIO stands for Quality Improvement Organization.

Concern for health data loss and misuse within the HIM Department requires that the health information practitioner evaluate all but which of the following? A. use of a back-up system B. policies and procedures developed to safeguard privacy and security C. salary for the database administrator D. security controls and access privileges of staff

C. salary for the database administrator **The database administrator's salary is irrelevant to the evaluation of health data loss and misuse.

Under the HIPAA Privacy Rule, when destruction services are outsourced to a business associate, the contract must provide that the business associate will establish the permitted and required uses and disclosures and include all but which of the following elements? A. method of destruction or disposal B. time that will elapse between acquisition and destruction or disposal C. the hospital's liability insurance in specified amounts D. safeguards against breaches

C. the hospital's liability insurance in specified amounts **The hospital's liability insurance is not included.

A primary focus of screen format design in a health record computer application should be to ensure that A. programmers develop standard screen formats for all hospitals. B. paper forms are easily converted to computer forms. C. the user is capturing essential data elements. D. data fields can be randomly accessed.

C. the user is capturing essential data elements. **Both paper-based and computer-based records share similar design considerations. Among these are the selection and sequencing of essential data items.

A HIM Department, currently using 2,540 linear filing inches to store records, plans to purchase new open-shelf filing units. Each of the shelves in a new 6-shelf unit measures 36 linear filing inches. It is estimated that an additional 400 filing inches should be planned for to allow for 5-year expansion needs. How many new file shelving units should be purchased? A. 12 B. 13 C. 11 D. 14

D. 14 **2,540 + 400 = 2,940 inches needed36 × 6 = 216 inches per unit2,940 (inches needed) divided by 216 (inches per unit) = 13.61 shelvesYou must buy 14 units because you cannot purchase a 13.61 filing shelf.

You have been appointed as chair of the Health Record Committee at a new hospital. Your committee has been asked to recommend time-limited documentation standards for inclusion in the medical staff bylaws, rules, and regulations. The committee documentation standards must meet the standards of both the Joint Commission and the Medicare Conditions of Participation. The standards for the history and physical exam documentation are discussed first. You advise them that the time period for completion of this report should be set at A. 12 hours after admission. B. 24 hours after admission. C. 12 hours after admission or prior to surgery. D. 24 hours after admission or prior to surgery.

D. 24 hours after admission or prior to surgery. **This time requirement complies with both Joint Commission and COP standards.

A data item to include on a qualitative review checklist of newborn inpatient health records that need NOT be included on adult records would be A. condition on discharge. B. chief complaint. C. time and means of arrival. D. APGAR score.

D. APGAR score. **"Chief complaint" and "time and means of arrival" are items that should be documented on any inpatient record. "Condition on discharge" reflects a data item you would expect to find on ER records. APGAR scores are part of newborn documentation.

Which of the following statements would be found in the laboratory report section of the health record? A. Morphine sulfate gr. 1/4 q.4h. for pain B. TPR recorded q.h. for 12 hours C. IV sodium Pentothal 1% started at 9:05 AM D. BUN reported as 20 mg

D. BUN reported as 20 mg **The blood urea nitrogen (BUN) test evaluates kidney function. The results of this test would be found on a laboratory report.

A 200-bed acute care hospital currently has 15 years of paper health records and filing space is limited. What action should be taken? A. Destroy records of all deceased patients. B. Return inactive records to each individual patient. C. Maintain the records indefinitely in hard copy. D. Destroy inactive records that exceed the statute of limitations.

D. Destroy inactive records that exceed the statute of limitations. **Before destroying health records, health care facilities must consider regulations, statutes, accreditation standards, organizational policies, pending litigation, storage, and cost.

Which one of the following is NOT an advantage of a computerized master patient index? A. It solves most space and retrieval problems. B. It allows access to data alphabetically, phonetically, or by date of birth, Social Security number, medical record, or billing number. C. It provides other departments with immediate access to the information maintained in the master patient index. D. Duplication of patient registration can never occur.

D. Duplication of patient registration can never occur. **Duplication of patient health records is a quality issue in the MPI. This is a disadvantage of the MPI.

For inpatients, the first data item collected of a clinical nature is usually A. expected payer. B. review of systems. C. principal diagnosis. D. admitting diagnosis.

D. admitting diagnosis. **Clinical data include all health care information collected during a patient's episode of care. During the registration or intake process, the admitting diagnosis, provided by the attending physician, is entered on the face sheet. If the patient is admitted through the ED, the chief complaint listed on the ED record is usually the first clinical data collected. The principal diagnosis is often not known until after diagnostic tests are conducted. Demographic data are not clinical in nature. The review of systems is collected during the history and physical, which is typically done after admission to the hospital.

When evaluating an outside contract with a paper record storage facility, all but which of the following are important factors to rate? A. cost B. emergency returns C. storage after filming D. cache memory

D. cache memory **Cache is used to store data on a computer.

In negotiating a contract with a commercial storage company for storage of inactive records, what would be the most important issue to clarify in writing? A. what are the billing terms B. who completes the list of what records are to be stored C. who will purge inactive files for transfer D. confidentiality policies and liability concerns

D. confidentiality policies and liability concerns **The health care facility must adhere to privacy and security regulations while records are in storage.

If there is more than one patient with the identical last name, first name, and middle initial, the master patient index entries are then arranged according to the A. Social Security number. B. date of admission. C. mother's maiden name. D. date of birth.

D. date of birth. **For confidentiality reasons, the patient's Social Security number should not be used. The names would appear in the MPI in the following order: Brown, Charlotte M. DOB: January 22, 1970Brown, Charlotte M. DOB: June 12, 1981Brown, Charlotte M. DOB: December 18, 1981

The HIM Department receives a request for a certified copy of a birth certificate on a patient born in the hospital 30 years ago. The Department should A. issue a copy of the birth certificate from the patient's record. B. issue a copy of the newborn's record. C. direct the request to the attending physician. D. direct the request to the state's office of vital records.

D. direct the request to the state's office of vital records. **Official birth records are maintained by the state.

In addition to diagnostic and therapeutic orders from the attending physician, you would expect every completed inpatient health record to contain A. standing orders. B. telephone orders. C. stop orders. D. discharge order.

D. discharge order. **Although many patient health records may feasibly contain all of the orders listed, only the discharge order is required to document the formal release of a patient from the facility. Absence of a discharge order would indicate that the patient left against medical advice and this event should be thoroughly documented as well.

Based on the following documentation in an acute care record, where would you expect this excerpt to appear? **Initially the patient was admitted to the medical unit to evaluate the x-ray findings and the rub. He was started on Levaquin 500 mg initially and then 250 mg daily. The patient was hydrated with IV fluids and remained afebrile. Serial cardiac enzymes were done. The rub, chest pain, and shortness of breath resolved. EKGs remained unchanged. Patient will be discharged and followed as an outpatient.** A. admission note B. physical exam C. clinical laboratory report D. discharge summary

D. discharge summary **The excerpt clearly indicates an overall summary of the patient's course in the hospital, which is a common element of the discharge summary.

Under the Patient Self-Determination Act of 1990, evidence of advance directives A. must be prepared by an attorney. B. is not required to be documented in the health record. C. requires a doctor's approval. D. is required to be documented in the health record.

D. is required to be documented in the health record. **According to the Patient Self-Determination Act of 1990, advanced directives are required to be documented in the health record.

Which of the following is NOT a data retrieval tool? A. color, animation, sound, icons B. screen design C. SQL D. light pen

D. light pen **Data cannot be retrieved with a light pen.

A health care facility has made a decision to destroy computerized data. AHIMA recommends which one of the following as the preferred method of destruction for computerized data? A. overwriting data with a series of characters B. disk reformatting C. overwriting the backup tapes D. magnetic degaussing

D. magnetic degaussing **Computerized data can be erased by neutralizing the magnetic field. This destruction method is called magnetic degaussing.

In preparation for an upcoming site visit by Joint Commission, you discover that the number of delinquent records for the preceding month exceeded 50% of discharged patients. Even more alarming was the pattern you noticed in the type of delinquencies. Which of the following represents the most serious pattern of delinquencies? Fifteen percent of delinquent records show A. absence of SOAP format in progress notes. B. missing discharge summaries. C. missing signatures on progress notes. D. missing operative reports.

D. missing operative reports. **"Missing signatures on progress notes" and "missing discharge summaries"—both signature omissions and discharge summary reports can be captured after discharge, but history and physicals should be on the chart within 24 hours of the patient's admission. The SOAP format is not a requirement of Joint Commission. Institutions are given a Type I recommendation when 2% of delinquent records are due to missing history and physicals or operative reports.

For continuity of care, ambulatory care providers are more likely than providers of acute care services to rely on the documentation found in the A. transfer record. B. interdisciplinary patient care plan. C. discharge summary. D. problem list.

D. problem list. **Patient care plans, pharmacy consultations, and transfer summaries are likely to be found on the records of long-term care patients.

What would be the most cost-effective and prudent course of action for the storage or disposition of 250,000 records at a large teaching and research hospital? A. purging and storing all death records off-site at a cost of $20,000 per year B. destroying all records older than 3 years for a cost of $50,000 C. storing the records off-site at a cost of $25,000 per year D. scanning all 250,000 records for a cost of $195,000

D. scanning all 250,000 records for a cost of $195,000 **Scanning the records would make them easily accessible and would not require storage costs over a long period of time.

The health care providers at your hospital do a very thorough job of periodic open record review to ensure the completeness of record documentation. A qualitative review of surgical records would likely include checking for documentation regarding A. whether a postoperative infection occurred and how it was treated. B. the quality of follow-up care. C. whether the severity of illness and/or intensity of service warranted acute level care. D. the presence or absence of such items as preoperative and postoperative diagnosis, description of findings, and specimens removed.

D. the presence or absence of such items as preoperative and postoperative diagnosis, description of findings, and specimens removed. **"Whether a postoperative infection occurred and how it was treated" represents an appropriate job for the infection control officer. "The quality of follow-up care" represents the clinical care evaluation process, rather than the review of quality documentation. "Whether the severity of illness and/or intensity of service warranted acute level care" is a function of the utilization review program. The correct answer is "the presence or absence of such items as preoperative and postoperative diagnosis, description of findings, and specimens removed."

A key data item you would expect to find recorded on an ER record but would probably NOT see in an acute care record is the A. instructions for follow-up care. B. physical findings. C. lab and diagnostic test results. D. time and means of arrival.

D. time and means of arrival. **All choices are required items in BOTH acute and ER records except "time and means of arrival," which is a required item only for ER documentation.

Health Informatics, Inc. is a vendor with a large collection of clinical information systems and hospital information systems that are designed to share data without human or technical intervention. This is a(an) A. integrated system. B. standard. C. interfaced system. D. OLAP.

interfaced system. **Interfaces make it possible for organizations to exchange data.


Conjuntos de estudio relacionados

IB Economics Chapter 11 - Macroeconomic objectives II: Economic growth and equity in the distribution of income"

View Set

Pulmonary Physiology & Pathology E3

View Set

6th Grade Math - Unit 7 - Bar Models + Algebraic Expressions

View Set

Personal Finance: Investing Basics

View Set

Art appreciation revel chapter 14

View Set

PrepUs for Pediatrics Chapter 29

View Set